Torts Flashcards

You may prefer our related Brainscape-certified flashcards:
1
Q

What are the elements for an assault?

A

Elements:

a. D must intend to place P in reasonable apprehension;
b. Of an immediate/imminent battery.

How well did you know this?
1
Not at all
2
3
4
5
Perfectly
2
Q

What element is common to all seven intentional torts?

A

Intent

Note***IIED can be intentional or reckless.

How well did you know this?
1
Not at all
2
3
4
5
Perfectly
3
Q

What does apprehension mean for an assault?

A

Apprehension means “knowledge”, not “fear”. P sees it coming, but is not necessarily afraid. A weakling can assault Brock Lesnar.

How well did you know this?
1
Not at all
2
3
4
5
Perfectly
4
Q

Describe the principle behind the “unloaded gun problem.”

A

D threatens, but cannot commit a battery because his gun is empty. If P has reason to believe it is loaded, then it’s assault. If P knows it’s empty, then it’s not an assault because he knows there will be no battery. An apparent ability creates a reasonable apprehension.

How well did you know this?
1
Not at all
2
3
4
5
Perfectly
5
Q

What is the meaning behind the word immediate or imminent in the elements for an assault?

A

Imminent

  1. Words alone lack immediacy. There must be conduct/menacing gesture, like displaying a weapon or raising a hand.
  2. Standing with hands in pocket, “In 30 seconds, I’m going to hit you, and I used to be a boxer.” – Not an assault. No reason to believe that the battery is imminent until the hands come out of the pockets. It is an idle threat until there is conduct.
  3. Words that accompany the gesture can neutralize the immediacy. For example:
    a. Conditional words – “If you weren’t my best friend, I’d beat you with this stick.” [holding above my head]. = No assault.
    b. “I’ll beat you with this stick tomorrow.” = No assault, not imminent.
How well did you know this?
1
Not at all
2
3
4
5
Perfectly
6
Q

What are the elements for a battery?

A

a. D must intend to and commit a harmful or offensive contact; and
b. That contact must be with P’s person.

How well did you know this?
1
Not at all
2
3
4
5
Perfectly
7
Q

What does it mean for something to be offensive for purposes of a battery?

A

Not permitted by a person with ordinary sensitivity, not hypersensitivity.

e.g. Tapping someone’s shoulder is not a battery; Stroking a random woman’s hair on the subway is a battery.

How well did you know this?
1
Not at all
2
3
4
5
Perfectly
8
Q

What is the extended personality rule?

A

Extended Personality Rule - P’s person includes anything P is holding or connected to at the time of the contact. E.g.:

  1. Purse
  2. Briefcase
  3. Clothing
  4. A horse you are riding
How well did you know this?
1
Not at all
2
3
4
5
Perfectly
9
Q

If you poison someone’s lunch, and they do not eat it for three hours. Battery?

A

Yes.

You can have a battery with a delay.

How well did you know this?
1
Not at all
2
3
4
5
Perfectly
10
Q

What are the elements for IIED?

A

a. D must engage in extreme and outrageous conduct; and
b. P must suffer severe emotional distress.

Note***Conduct can be intentional or reckless.

How well did you know this?
1
Not at all
2
3
4
5
Perfectly
11
Q

What conduct will be considered outrageous for purposes of an IIED claim? What are the hallmarks?

A
  1. Exceeds all bounds of decency tolerated in a civilized society.
  2. Insults/cursing alone do not meet this test unless the person is a member of a fragile class.
  3. Hallmarks of outrageousness:
    a. Conduct is continuous or repetitive in nature (harassment). E.g. Abusive debt collection.

b. D is a common carrier or innkeeper. E.g. An employee of one of these places does something intentionally to upset you.

c. P is a member of a fragile class of persons.
i. Little kids/Young children.
ii. Elderly persons.
iii. A known pregnant woman.

d. D has prior knowledge of P’s emotional weakness/sensitivity, and D deliberately targets it.

How well did you know this?
1
Not at all
2
3
4
5
Perfectly
12
Q

What does it mean to have severe distress for an IIED claim?

A
  1. No specific element required to be shown.
  2. Don’t have to show physical symptoms or missed work, for example.
  3. The most common way this is tested is that the examiners will negate the problem by stipulating to the opposite.
    a. E.g. Long problem about D’s behavior. Near the end, P was mildly annoyed by this conduct. Mild annoyance is not going to get you IIED.
How well did you know this?
1
Not at all
2
3
4
5
Perfectly
13
Q

What are the elements for a false imprisonment?

A

a. D must intentionally commit an act of restraint; and

b. P must be confined in a bounded area.

How well did you know this?
1
Not at all
2
3
4
5
Perfectly
14
Q

What is an act of restraint for purposes of false imprisonment?

A
  1. Threats can be sufficient to restrain.
    a. “If you leave this room, I’ll shoot you. I’ll be right down the hall watching.” [I’m holding a gun.] = restraint.
    b. Hypersensitivity is not taken into account. E.g. “I’ll turn you into a unicorn if you leave with my magic want.” [Person believes you have a magic wand, and it works.] = No restraint.
  2. Obviously can be physical as well.
  3. An omission can be an act of restraint if D owed P a pre-existing duty. E.g. Leaving someone in a wheelchair who is relying on you to push them is an omission that amounts to a restraint.
  4. P must know of the act of restraint or be harmed by it. - I lock you in your room while you’re asleep, but unlock it before you wake up, and no harm results while it was locked. = No restraint.
How well did you know this?
1
Not at all
2
3
4
5
Perfectly
15
Q

What is a bounded area for purposes of false imprisonment?

A
  1. Does not have to be specific/precise boundaries. E.g. Detained by store security guard who says “Wait here.”
  2. The area does not have to have fences or walls. E.g. “I’ll shoot you if you leave your street.”
  3. An area is not a bounded if there is a reasonable means of escape that P can reasonably discover. That is, you’re not locked in, if you can get out.
    a. E.g. P confined to a space, and the way out is dangerous, disgusting, or humiliating, it is not a reasonable means of escape.
    b. E.g. If the way out is hidden, it is not a reasonable means of escape.
How well did you know this?
1
Not at all
2
3
4
5
Perfectly
16
Q

What are the elements for a trespass to land?

A

a. An act of physical invasion; and

b. That act must interfere with P’s exclusive possession of the property.

How well did you know this?
1
Not at all
2
3
4
5
Perfectly
17
Q

What exactly is an act of physical invasion for the purposes of a trespass to land?

A
  1. Can have the act of physical invasion if D enters the property.
    a. D need not intend or even be aware that he crossed the boundary line.
    b. He only needs to intend to put one foot in front of the other.
  2. Throw something on the land
  3. Intangible invasions are not trespass (could be nuisance)
    a. Odors
    b. Light
    c. Noises
How well did you know this?
1
Not at all
2
3
4
5
Perfectly
18
Q

What does it mean to interfere with possession for the purpose of a trespass to land?

A
  1. The tort belongs to the person in possession of the property, not the owner.
  2. Your right to the property is not limited to the surface. It includes the air space above and soil below to a reasonable point from the surface.
How well did you know this?
1
Not at all
2
3
4
5
Perfectly
19
Q

What are the common elements for a trespass to chattels and conversion?

A

Intentional interference with personal property

i. D can either deliberately damage your personal property; or
ii. D can take your property from you (steal it).
iii. These are your private civil damages for vandalism and theft.

Note***The chattel need not be completely destroyed nor does it mean that the owner was permanently deprived of it. All that is required is a serious invasion. Therefore, if a bailee lends the bailor’s car to a third party without the bailor’s consent, then the bailor comes to get the car only to discover it is missing, the bailee is liable for conversion even if the car is later returned safely.

How well did you know this?
1
Not at all
2
3
4
5
Perfectly
20
Q

What is the difference between trespass to chattels and conversion?

A

If the amount of interference is small = trespass to chattels.
If the amount of interference is significant = conversion

Conversion has special remedies

i. Full market value of the item (like a forced sale); or
ii. Replevin (return) – lecturer didn’t mention, but the outline does.

How well did you know this?
1
Not at all
2
3
4
5
Perfectly
21
Q

What affirmative defenses may be available to intentional torts?

A

1) Consent
2) Self Defense; Defense of Others; Defense of Property
3) Necessity

How well did you know this?
1
Not at all
2
3
4
5
Perfectly
22
Q

What is required to claim consent as a defense?

A

Consent – Available to all 7 Intentional Torts

i. P must have legal capacity to give consent.
1. Age might not allow consent (unless it is an age appropriate invasion) e.g. an 11 year old can consent to wrestle with his other 11 year old buddy.
2. Intoxication might not allow for consent.
3. Mental disability might not allow for consent.

ii. Express consent – Explicit declaration by P that grants D permission to do something.
1. Not often tested b/c it’s easy unless they’re testing the exception.
2. Exception - Express consent is void if it is obtained through fraud or duress. E.g. Telling someone you are a doctor in order to get consent to examine that person when, in fact, you are not a doctor.

iii. Implied consent
1. Customary practice/Custom and usage - Person participates in an activity where such invasions are normal. E.g. playing sports.
2. D’s reasonable interpretation of P’s objective conduct and the surrounding circumstances. E.g. the right to read the situation and act according to well observed social norms.

iv. All consent has a scope
1. D must remain in the scope of the consent.
2. If D leaves the scope, then he is back to intentional tort territory.
3. E.g. In basketball, you consent to being fouled in a normal way. You do not consent to being purposefully punched in the face.
4. E.g. If you consent to being operated on your knee, and the surgeon gives you a rhinoplasty. That’s a battery.

How well did you know this?
1
Not at all
2
3
4
5
Perfectly
23
Q

What is required to claim one of the protective privileges, that is, self defense, defense of others, or defense of property?

A

i. In all 3, D must have proper timing.
1. Only privilege if the threat D is responding to is in progress or imminent.
2. No pre-emption. E.g. beating someone up today because they threatened to beat me up tomorrow.
3. No Revenge. E.g. I got beat up 30 mins ago. I go track the guy down and beat him up.
4. If someone hits you, and that person is still standing in front of you, and you have a genuine reason to believe that another blow is coming. You may hit back.

ii. Must limit yourself to proportional or necessary force.
1. Excessive force in response is a tort.
2. An imminent slap in the face allows me to grab their arm or shove them or hit them first. You cannot stab them or shoot them.
3. In a life threatening situation, you may resort to deadly force.
a. You may not use deadly force to protect your property.
b. You can’t shoot someone for stealing your laptop.
c. You can’t set up a deadly booby trap (spring gun case).

iii. Self-Defense not available if you are the initial aggressor unless the other party escalates.
iv. Majority says no duty to retreat, but modern trend says you must retreat before using deadly force if it is safe to do so, and you are not in your home.
v. Defense of property only allows force to stop it while it occurs or while in hot pursuit. Furthermore, the Shopkeeper’s Privilege may detain a person he reasonably believes is a thief, in a reasonable manner, for a reasonable time for the purpose of conducting an investigation.

How well did you know this?
1
Not at all
2
3
4
5
Perfectly
24
Q

What are the two types of necessity?

A

1) Public

2) Private

How well did you know this?
1
Not at all
2
3
4
5
Perfectly
25
Q

Under what circumstances can D avail himself of public necessity?

A

Public necessity – D commits a property tort in an emergency to protect the community as a whole or a significant group of people.

  1. E.g. a fire is being spread by strong winds, and it is burning homes and reaching a gasoline storage facility that will kill people if it blows up. The community savior breaks into a warehouse to steal flame retardant chemicals and puts out the fire. Savior is not liable for his trespasses.
  2. This is an absolute defense. It is absolute so that the good Samaritan will not hesitate.
How well did you know this?
1
Not at all
2
3
4
5
Perfectly
26
Q

Under what circumstances can D avail himself of private necessity?

A

Private necessity – In an emergency, D commits a property tort to protect his own interest.

  1. D is not a good Samaritan.
  2. Not an absolute defense. 3 consequences.
    a. D is liable for actual harm to the property, that is, compensatory damages.
    b. D is immune from nominal and punitive damages.
    c. As long as the emergency continues, P must tolerate D’s presence on the land.
How well did you know this?
1
Not at all
2
3
4
5
Perfectly
27
Q

Dave is cross country hiking. Dave is in rural northern MN in February. An unexpected blizzard rolls in. Temps drop 30 degrees in 15 minutes. Snow and wind produce white out conditions. Dave will die if he doesn’t find shelter. He sees a lone farmhouse. No one is home so he breaks in and leaves after the storm. What result?

A

Owner can get money from the hiker for the broken lock.

How well did you know this?
1
Not at all
2
3
4
5
Perfectly
28
Q
  1. Base fact pattern: Dave is cross country hiking. Dave is in rural northern MN in February. An unexpected blizzard rolls in. Temps drop 30 degrees in 15 minutes. Snow and wind produce white out conditions. Dave will die if he doesn’t find shelter. He sees a lone farmhouse. The door is unlocked so Dave goes in and leaves after the storm.
A

Owner has no damage because nothing was broken, and he cannot sue for the trespass because Dave was privileged under private necessity.

How well did you know this?
1
Not at all
2
3
4
5
Perfectly
29
Q
  1. Base fact pattern: Dave is cross country hiking. Dave is in rural northern MN in February. An unexpected blizzard rolls in. Temps drop 30 degrees in 15 minutes. Snow and wind produce white out conditions. Dave will die if he doesn’t find shelter. He sees a lone farmhouse.The door is unlocked so Dave goes in. Owner comes downstairs and pushes him out the door.
A

Owner is liable for any harm that comes to Dave.

How well did you know this?
1
Not at all
2
3
4
5
Perfectly
30
Q

I tie up my boat on your dock during a storm so that I do not capsize my boat and get tossed into the sea. The next day, there is some damage to the dock from my boat. What result?

A

I was entitled to stay until the storm ended. I had “sanctuary”. But, if me tying up my boat damaged your dock. I have to pay for the damage to your dock.

How well did you know this?
1
Not at all
2
3
4
5
Perfectly
31
Q

What is defamation? What are the elements?

A

A statement that tends to adversely affect reputation.

1) D must make a defamatory statement
2) Specifically identifying P (of and concerning P).
3) Publication – Misleading because it is more than print.
4) Damages

In MA, Defamation against a private person:
The elements above plus:
1) falsity; and
2) negligence.

How well did you know this?
1
Not at all
2
3
4
5
Perfectly
32
Q

Describe what qualifies as a defamatory statement, specifically, identifying P.

MA standard?

A

a. An allegation of fact; and

b. That allegation of fact, reflects negatively upon someone’s character so people think badly about the person discussed.
i. John Anderson is embezzling money from the company.
ii. Dr. Mark Baker carelessly killed three people during surgery this month.
iii. Steve Clark is molesting children at the day care center where he works.

c. Name calling is not defamatory. E.g. John Brady is a son of a bitch.

d. P doesn’t have to be identified by name.
i. The US Attorney for the District of MA has deliberately destroyed exculpatory evidence to get convictions of innocent people. That IDs P.
ii. If you give a name, but the name is super common, it does not satisfy the element. Michael Smith could be a lot of people.

e. No liability for a deceased person. The person must be alive to recover.

MA Standard = discredits P in the minds of any considerable and respectable class in the community.

How well did you know this?
1
Not at all
2
3
4
5
Perfectly
33
Q

What does “Publication” mean in a defamation suit?

A

Publication – Misleading because it is more than print.
a. Telling you something about you in a one on one situation will not result in defamation because it will not hurt your reputation.

b. This is a de minimus element. D shares the statement with one person other than P. That is publication.
c. The more people that are told, the greater the harm, and the greater the damages.

d. Publication does not have to be deliberate. Negligence counts.
i. I mistakenly leave the intercom on when I say something.
ii. My office is wiretapped without my knowledge = not publication. Not my negligence.
iii. It was reasonably foreseeable that the speaker’s comment would be overheard.

How well did you know this?
1
Not at all
2
3
4
5
Perfectly
34
Q

What damages are needed in a defamation suit?

A

a. Assumed in many, many cases, but not all.
b. Damages are presumed in a libel case. Libel involves an underlying statement that is written down or captured in a permanent format like taped, filmed, recorded.
c. Damages are not always presumed in slander. Slander involves a spoken statements.

i. Damages are presumed in slander per se cases.
1. Statement relates to P’s business or profession. Dr. Mark Baker killed three people in surgery this month.
2. Statement that P committed a serious crime (a crime of moral turpitude/violence or dishonesty). Bob Davis killed someone.
3. Statement that imputes un-chastity to a woman. Suzy Davis is a slut.***Not recognized in MA.
4. Statement that P suffers from a loathsome disease.
a. Leprosy – Won’t be on the exam.
b. Venereal disease – That one might show up.

ii. Damages must be proven in all other slander cases.
1. E.g. economic harm from being fired or people stopped coming to your restaurant.
2. Social harm is not enough. The fact that people don’t want you on the softball team or don’t invite you to parties is not enough.

How well did you know this?
1
Not at all
2
3
4
5
Perfectly
35
Q

What affirmative defenses are available in defamation cases?

A

1) Express or Implied Consent
2) Truth
3) Absolute Privilege
4) Qualified Privilege

How well did you know this?
1
Not at all
2
3
4
5
Perfectly
36
Q

Who has the burden to prove truth in a standard common law defamation suit?

A

If D can prove that his statement, which hurt P, was true, D is not on the hook for damages. D has the burden of proof.

How well did you know this?
1
Not at all
2
3
4
5
Perfectly
37
Q

What are the absolute privileges in a defamation case?

A

Absolute Privileges

  1. Spouses – When spouses speak to each other, they are immune from defamation for statements made to one another about a third person.
  2. Gov’t officials in the conduct of their official duties. Extends to judges, lawyers, and witnesses when acting in connection with judicial procedures (pleadings, motions, trials, etc.) cannot be the basis for a defamation suit. Congress’ statements during a committee hearing. Etc.
How well did you know this?
1
Not at all
2
3
4
5
Perfectly
38
Q

What is a qualified privilege in a defamation suit?

A

Qualified Privileges – Applies when there is a public interest in encouraging candor. The most common example is recommendations and References – e.g. professional references, housing references, creditor references. These are only valuable if they are honest.

  1. To keep the privilege
    a. D must have an honest and reasonable belief that his statement was accurate; and
    b. D must confine himself to relevant material. Do not inject extraneous stuff.
How well did you know this?
1
Not at all
2
3
4
5
Perfectly
39
Q

How does defamation analysis change if it is a matter of public concern? Any MA distinctions?

A

What if the defamatory statement relates to a matter of public concern?

a. There is widespread interest. A journalist would find it newsworthy.
i. Dishonest behavior by politician.
ii. Athlete taking PEDs.
iii. Military officer is selling secrets to enemy.

b. 2 Extra elements
i. P must prove falsity. Truth is presumed unless P shows falsity.
ii. If P is a public figure, P must show actual malice (D knew it was false) or recklessness (D did not do anything to check accuracy of statement.)
iii. If P is a private figure, P need only show that D was negligent in finding out whether his statement was correct.

c. Bottom line: A reasonable and honest mistake on a matter of public concern, will not result in liability.

Note***MA distinctions

a. All Ps must prove negligence. An honest mistake is always a defense.
b. By statute, there are no punitive damages available in defamation actions.
c. MA recognizes a separate claim for commercial disparagement when you knowingly/recklessly publish a false statement about that business’ products or services with the intent to damage the business and such harm results.
d. There is a similar claim for negligent misrepresentations made in the course of business involving a careless false misrep. causing a loss to others relying on your misrep. (see outline)

How well did you know this?
1
Not at all
2
3
4
5
Perfectly
40
Q

What are the four privacy torts?

A

1) Appropriation
2) Intrusion*
3) False Light**
4) Disclosure***

  • MA has banned wiretapping and allows for civil action.
    • Not recognized in MA
  • **Clearly recognized in MA
How well did you know this?
1
Not at all
2
3
4
5
Perfectly
41
Q

What is appropriation?

A

Appropriation – “To take something that doesn’t belong to you and appropriate it.” D used P’s name or image for commercial purposes.

a. E.g. Cereal company puts TB12’s pic on a cereal box w/o permission.
b. Remedies = injunction or damages.
c. If purpose of use isn’t pure commerce, but a newsworthy element, then it is not actionable. E.g. SI can put Brady’s pic on the cover.
d. Cause of action is not limited to celebrities, but it usually involves celebrities.

How well did you know this?
1
Not at all
2
3
4
5
Perfectly
42
Q

What is intrusion?

A

Intrusion – Invasion of P’s seclusion in a way that is highly offensive to an average person.

a. E.g. Wiretapping, covert video surveillance, intercepting emails, peeping Toms, listening at the key hole.

b. You have to be in a place where you have REP.
i. Eavesdroppers at a cocktail party do not count as Ds.
ii. Snapping a photo while you are on the street is not actionable.

How well did you know this?
1
Not at all
2
3
4
5
Perfectly
43
Q

What is False Light? MA distinction?

A

3) False Light – Widespread dissemination of a material falsehood about P that would be highly offensive to an average person.
a. MA does not recognize this claim
b. Can be deliberate, running around town telling a lie.
c. Could also be an inadvertent juxtaposition. E.g. a photograph that accompanies a news story about dishonest cab drivers. Use a stock photograph of cab driver taking a break. That cab is actually an honest person. That photo is placing that cab driver in a false light.
d. Using a photo of a politician that makes it look like he is coming out of a brothel, when, in fact, he was not.
e. This can overlap with defamation. Pete Moss was embezzling money from the company. This is both defamation and false light.
i. Economic damages from defamation.
ii. Social damages from false light.

How well did you know this?
1
Not at all
2
3
4
5
Perfectly
44
Q

What is disclosure?

A

Disclosure – Widespread disclosure of confidential information about P, which is highly offensive to an average person. This is essentially “Gossip.”

a. E.g. The receptionist at my doctor mails my medical records to everyone on my street.
b. Exception – A newsworthy exception.
i. The Globe publishing my medical records is a tort.
ii. The Globe publishing Hilary Clinton’s medical records is not a tort.

How well did you know this?
1
Not at all
2
3
4
5
Perfectly
45
Q

What two torts should I go look up in the CMR, but not devote much time to?

A

1) Fraud

2) Malicious Prosecution

How well did you know this?
1
Not at all
2
3
4
5
Perfectly
46
Q

What are the four elements of negligence?

A

1) Duty
2) Breach
3) Causation
4) Harm

How well did you know this?
1
Not at all
2
3
4
5
Perfectly
47
Q

2 guys are rushing to catch a train and try to jump through an open door. An employee shoves them through an open door, and, in doing so, a package in their hands is knocked loose. The package had fireworks, so when it fell, it exploded, and Helen Palsgraf was injured.

Does she have a claim? Why/why not? What if a citizen was injured at the scene of the accident while trying to render aid?

A

A duty is owed to foreseeable victims. Unforeseeable victims always lose.

  1. Cardozo View – Helen Palsgraf was outside the zone of danger, and she was an unforeseeable victim.
  2. Simple View – Helen Palsgraf lost because she was way the hell out of the way.

Rescuers are allowed to recover even if they started way the hell out of the way because danger invites rescue. (Firefighters, police, EMTs, etc. cannot recover for facing the inherent risks of their duties, except in MA.)

How well did you know this?
1
Not at all
2
3
4
5
Perfectly
48
Q

What is the default standard of care in a negligence case?

A

You must exercise the same level of care as a reasonably prudent person in like circumstances. = human buzzkill; Ned Flanders.

Objective standard of care. Everyone must live up to it. Unless it is displaced.

How well did you know this?
1
Not at all
2
3
4
5
Perfectly
49
Q

D kept soaked kerosene rags in his garage all summer. It ignited and burned down P and D’s house (he lived next door). P sues. D says, “I’m uneducated, and I have a really low IQ.”

A

Doesn’t matter. Held to the standard of a reasonable person in similar circumstances.

How well did you know this?
1
Not at all
2
3
4
5
Perfectly
50
Q

In what circumstances will defendants be held to a standard other than the general reasonable person standard?

A
  1. Superior Skill/Knowledge
  2. Disability
  3. Kids
  4. Professionals
  5. If D has superior skill or knowledge, the standard elevates to the level of a reasonable person with that skill or knowledge.
    a. E.g. Skill of a race car driver
    b. E.g. Knowledge of a dangerous intersection.
  6. D’s physical characteristics are held to the standard of a reasonable person with that disability.
    a. E.g. D is blind.
    b. E.g. D is in a wheelchair.
    c. Sometimes it is relevant, but it would be irrelevant to storing gasoline soaked rags in a garage all summer.
  7. Kids
    a. Very young children owe the world zero duty of care. Generally, under 5.
    b. Children over that age must exercise a level of care of similar to a child of similar age, experience, and intelligence acting under similar circumstances.
    c. Little Billy (age 6) riding his bike down to the corner and back. It is his first time, and he is of below average intelligence. Suzy is sitting on the pavement playing jax. Little Billy rides his bike over Suzy’s hand and she sues Billy. He must operate his bike like a similarly dumb 6-year-old who has never ridden a bike before. It is very defendant friendly.
    d. Exception: If a child is engaging in an adult activity, then the child is judged with the adult reasonable person standard. This includes operating a vehicle with a motor. (Car, boat, jet ski, farm equipment, etc.)
  8. Negligence claims against professionals (malpractice)
    a. Covers lawyers, accountants, architects, doctors, etc.
    b. A professional owes her clients/patients the care of an average member of the same profession.
    c. Must go look and see what others in the same line of work actually do. That is, it is empirical. It is a fact based standard.
    d. It is a doctrine that demands that professionals conform to the standards of the practice. This because, in their collective wisdom, it is assumed that the profession has developed the best way of doing things.
    e. The custom of the profession sets the standard of care. Do what is customarily done.
    f. Requires expert testimony to educate the finder of fact on the standard of care.
    g. Modern trends = national standard.
How well did you know this?
1
Not at all
2
3
4
5
Perfectly
51
Q

Describe the duties of care for land occupier liability for the MBE?

A

Premises Liability/Land Occupier Liability – Person enters property and encounters a concealed dangerous condition.

a. Unknown Trespasser
i. Standard = No duty of care.
ii. Unknown trespasser always loses.
iii. An unknown trespasser is an unforeseeable victim.

b. Known/Anticipated Trespasser
i. There has been a pattern of past trespassing so owner should expect it to continue.
ii. Standard = protect trespasser if it is a known man-made death trap. Broken down:
1. Artificial condition (constructed by humans). A natural condition includes snow and ice = not manmade.
2. The condition is highly dangerous (could kill or seriously injure)
3. Hazard is concealed (not open and obvious)
4. Possessor knew about it in advance.

c. Licensee – e.g. social guests, recreational users, a politician getting signatures, Jehovah’s witnesses, etc.
i. Enters w/ permission but not conferring me an economic benefit.
ii. Standard = protect licensee from all known traps. Broken Down:
1. Condition concealed from licensee.
2. Possessor knew about it beforehand.
3. No duty to inspect.

d. Invitee – e.g. customer of a business, museum patron, picking up someone at the airport, church patron.
i. Enter w/ permission and confers an economic benefit on the landowner or the land is publicly open.
ii. Standard = Protect invitees from reasonably knowable traps. Broken down:
1. Condition concealed from the invitee.
2. Condition the possessor knew about in advance or could have discovered through a reasonable inspection. “Knew or should have known.”
a. A reasonable inspection is one that a reasonably prudent person would do.
b. Frequency and thoroughness varies.

How well did you know this?
1
Not at all
2
3
4
5
Perfectly
52
Q

What is the MA standard for land occupier liability?

A

MA has abolished the MBE approach in favor of a streamlined approach.

i. Lawful entrants – Licensees and Invitees.
1. All entitled to a duty of reasonable prudence under the circumstances.

ii. Unlawful entrants – Trespassers.
1. No duty of care, except to refrain from reckless behavior and an ordinary duty of care to aid/prevent injury to a trespasser whom the landowner knows to be in peril.
2. Amounts to same as multistate for known trespassers.
iii. Third category by statute = Recreational users who do not pay.
1. Only entitled to protection from gross negligence.

In MA, if a certain type of harm is foreseeable because of the type of business that is being run, the owner may be held liable if P can show he failed to take all reasonable precautions necessary to prevent the foreseeable dangerous condition.

How well did you know this?
1
Not at all
2
3
4
5
Perfectly
53
Q

What is the firefigher’s doctrine? How does it change n MA?

A

Firefighters, police, EMTs can never recover for things that are inherent risks of their job.

No firefighter rule in MA by statute. Allows these actors to recover.

How well did you know this?
1
Not at all
2
3
4
5
Perfectly
54
Q

What is the attractive nuisance doctrine?

A
Trespassing Kid(s) – Attractive Nuisance Doctrine
i.	Landowners owe a duty of reasonable prudence if they know or should know of a dangerous artificial condition on the land, when they should expect children, who cannot appreciate the risk, and the expense to remedy is slight compared with the magnitude of the risk.

ii. Should I have expected kids to come on my land?
1. Location
2. Is there an attractive nuisance?
a. Swing sets
b. A swimming pool
c. A junkyard with a hill of truck tires

Applies in MA.

How well did you know this?
1
Not at all
2
3
4
5
Perfectly
55
Q

How can a land owner protect himself from liability?

A

In any case land occupier liability case, except for trespassing children, you can do two things to protect yourself.

i. Fix the problem.
OR
ii. Give an adequate warning so the concealed hazard becomes an open hazard and satisfies your duty.
1. Could be a sign.
2. Could be caution tape.
3. Could be verbal.
4. You may have to remind them depending on circumstances. Telling them once, may not cut it.

How well did you know this?
1
Not at all
2
3
4
5
Perfectly
56
Q

What is negligence per se? Does it change in MA?

A
Negligence Per Se – Conclusive evidence of a breach.  Class of person, class of risk.
Still must provide causation and breach.

i. D violated a statute
ii. P is a member of the class meant to be protected by the statute.

iii. The harm is the type the statute was enacted to prevent.
E.g. Motor vehicle statutes, but that’s too easy for the bar exam.

Exceptions

  1. If statutory compliance is more dangerous than a violation, the statute is not used.
    a. E.g. crossing double yellow line to avoid a pedestrian in the road.
  2. Compliance would have been impossible under the circumstances.
    a. E.g. the motorist who had been completely healthy until now has a heart attack while driving.

In MA, even when the two-part test is met, and the statute comes in, it is only EVIDENCE of negligence. It does not automatically satisfy duty and breach.

How well did you know this?
1
Not at all
2
3
4
5
Perfectly
57
Q

Pot smoker lights up and, due to his stove leaking gas all day, the house blows up and damages his neighbor’s house. Can the neighbor use a misdemeanor marijuana possession statute to establish NPS?

A

NO! The intent of the statute was not to prevent neighbors from smokers blowing up their house and damaging yours. Fails both elements of the test. P is not penalized for failing NPS, he simply must litigate based on the normal standard of care.

How well did you know this?
1
Not at all
2
3
4
5
Perfectly
58
Q

Is there a duty to rescue?

A

Not generally.

a. They’ll test on it by giving a fact pattern where D acts like an asshole, but he still doesn’t have a duty to rescue.
b.
c. 2 Exceptions
i. Special Pre-Existing Relationship – there is a duty to act.
1. One of a formal, legal nature.
2. E.g. Innkeeper-guest, common carrier-passenger, business-invitee.
ii. If D put P in peril/Is the cause of the emergency.
1. Even if the peril wasn’t due to your negligence.
2. E.g. I skid on black ice while driving carefully, and I hit a pedestrian. I must help.
3. Imposes a duty to act, not a duty to rescue.

How well did you know this?
1
Not at all
2
3
4
5
Perfectly
59
Q

A, B, and C are Olympic swimmers, and they are walking down the street. They see a cute little girl drowning in a lake. She calls to them for help, and they think about helping, but they decide to walk away. Little girl drowns. A, B, and C liable?

A

Not liable. No duty to rescue.

How well did you know this?
1
Not at all
2
3
4
5
Perfectly
60
Q

Corey is standing with Rachel on a beach. As it turns out, they are standing on the edge of quicksand. He hears a noise and quickly turns. As he turns, he bumps Rachel into the quicksand. Does Corey owe Rachel a duty to act?

A

Yes. He put her in peril. It does not matter that he did not act negligently. His actions created the current risk to her well-being.

61
Q

What duties result from undertaking a rescue? Changes in MA?

A

a. If you begin to rescue, you cannot quit because others may have done nothing because you are on top of it.
b. If you choose to rescue, and you mess up, you are liable for your negligence.

Good Samaritan Law in MA

i. In MA, it only covers persons with professional training.
ii. Physicians, PAs, nurses, on duty certified medical technicians, police, and firemen.
iii. In MA, any person who in good faith attempts to render emergency care and does so without compensation will not be liable for acts or omissions other than gross negligence or wanton or willful misconduct resulting from such emergency care.

62
Q

When may a plaintiff recover for NIED?

A

3 Scenarios

1) Near Miss Cases
2) Bystander Cases
3) Relationship Cases

a. Near Miss Cases (Fright)
i. Negligent D – violated some other standard
ii. That D did not physically harm P (but he almost did)
iii. Must Prove:
1. D’s negligent conduct placed him in a zone of physical danger.
2. Suffered subsequent physical manifestations of that distress.
iv. Driver almost hits P, a pedestrian, when D was texting while driving. P was so startled that he had a heart attack. Injuries could also be a rash, a tremor, miscarriage, etc.

b. Bystander Cases (Grief)
i. D seriously injures or kills X.
ii. This leaves P very sad.
iii. Must prove:
1. P and X are close family members.
2. P was at the scene
3. P saw the injury as it happened (a contemporaneous witness.)
iv. You have a ring side seat to your loved one’s demise.
v. Spouse, parent, child definitely count.
vi. MA has relaxed the rule. If you come on the scene while X is still there or get to the hospital right after the accident, you may still recover.

c. Relationship Cases - involve a business relationship and some private enterprise.
i. Distress is highly predictable if the business messes up.
ii. E.g.
1. Medical patient/medical lab – The lab tells you that you have some terrible disease, but they screwed your sample up.
2. Customer/Funeral Parlor – The parlor mishandles the corpse. (e.g. lose the corpse, screw up the embalming, cremate when wanted a burial.)

63
Q

How may D breach his duty?

A

P must identify specific wrongful conduct or RIL.

a. In an essay I will write, “P will allege that the breach here was [this act or omission]. This is unreasonable because [of these reasons].”
b. Hand formula – Don’t use it unless there are numbers in the problem. Burden is less than magnitude x probability.
c. RIL – P lacks enough information to specifically allege the breach.
i. E.g. Barrel flies out of a bakery window and hits a pedestrian on the head.
ii. Use it to get to jury w/o breach.

iii. Must show:
1. Accident is the type of event that is normally does not happen unless there is negligent conduct. (Properly handled barrels don’t fly out the window.)
2. Negligence is attributable to D. This is usually, but not always, shown by demonstrating that the thing that caused the injury was in D’s exclusive control. Could also show by demonstrating a design defect. This shows the finder of fact that you are suing the right guy.

iv. This gets you to the jury. So if either party asks for a directed verdict, it should be denied.
v. Even if you prove RIL, you still must prove causation and damages.

64
Q

What are the two sub-elements of causation?

A

1) Cause in fact

2) Proximate Cause

65
Q

What is cause in fact?

A

Cause in fact – When you write about this, remember that D is not the factual cause, the breach is the factual cause. Also say that the breach was “a” factual cause, not “the” cause.

i. “But for” cause – But for the breach, P would not have been harmed.

ii. Joint Causes – Merged causes
1. Substantial Factor Test - Both breaches are enough to be responsible for the injury on their own, but they both breached.
a. 2 people start fires that merge, and the fire burns down P’s house.
b. 2 companies dump poisonous chemicals into a reservoir used for drinking water. Each dumps enough chemicals to poison people, but they both did it.

iii. Alternative Causes – Unascertainable Cause
1. Two people breached, but we don’t know which one actually caused the harm.
2. Summers v. Tice
3. Each D must prove that he is not actually responsible, or they will be held jointly liable.

66
Q

What is proximate cause?

A

b. Proximate Cause (it’s a “Fairness” check)
i. Measured in terms of foreseeability.

ii. Dave, bodyguard of AD FF, misses Franz’ motorcade. Had he been there, he would have seen the assassin coming, but he was not there and AD FF is killed. This kicks of WWI, the resulting treaty gives rise to Hitler, and Hitler commits genocide. Dave is liable for death of Arch Duke FF, but not Hitler genocide in WWII.

iii. Factors to determine foreseeability
1. Close in time?
2. Close in proximity?
3. Intermediate steps?

iv. Four scenarios that get tested frequently and are examples of proximate cause.
1. Intervening negligent medical treatment
2. Intervening negligent rescue
3. Intervening protection and reaction forces
4. Subsequent disease or accident.

v. If examiners do not use these four scenarios, then they will condition their answer. Yes, if the harm was foreseeable.

67
Q

Pedestrian is hit by negligent driver, breaks his leg, and is taken to the hospital. While at the hospital, the hospital puts a caste on way too tight. Weeks later, they must amputate his leg because of the tight caste. Is the driver the proximate cause of the amputation?

A

Yes.

“Intervening negligent medical treatment”
The driver is the proximate cause for the amputation. When he drove negligently and hit D, he drove P into the hands of the medical professionals, and medical malpractice is foreseeable. This does not relieve the Dr. from liability, but D is also liable.

68
Q

Pedestrian is hit by negligent driver, breaks his leg. A good Samaritan responds on the scene, and tries to drag P out of the street. In dragging him, the good Samaritan dislocates P’s shoulder. Is driver the proximate cause for P’s dislocated shoulder?

A

Yes.

“Intervening negligent rescue”

Driver is on the hook for P’s dislocated shoulder because danger invites rescue, and therefore, this scenario was foreseeable. Does not relieve the rescuer if the rescuer also acted negligently.

69
Q

Driver runs a red light while people are in a busy crosswalk. There are dozens of pedestrians in the crosswalk. When the car is running them down, the people freak out and stampede. P is trampled in the stampede. Is the driver the proximate cause of P getting trampled?

A

Yes.

“Intervening protection and reaction forces.”

The driver is on the hook for the P injured in the stampede because that result was foreseeable. Doesn’t relieve the stampeder if he was also acting negligently.

70
Q

Pedestrian is hit by negligent driver, breaks his leg, and is taken to the hospital. P has a good doctor, and is put on crutches, but he has never used them before. He falls and breaks his arm. Is the driver the proximate cause of P’s broken arm?

A

Yes.

“Subsequent disease or accident.”

D is on the hook for the fall because that was foreseeable. Don’t forget to look for contributory/comparative negligence.

71
Q

What is the Eggshell Skull Rule?

A

You are liable for all damages suffered by P because of your negligence even if P is significantly greater in scope than a normal person would have suffered. You take your plaintiff as you find him.

72
Q

What is comparative negligence? How does it work on the MBE? How does it work in MA?

A

1) P failed to exercise care for her own safety.
2) Duty to protect yourself & obey statutes that are self-protective in nature.
3) If P has fallen short, then jury must weigh the carelessness of each party. Jury then must assign a percentage to each party.
4) P’s recovery will be reduced depending on P’s percentage of fault.

5) MBE = Pure comparative negligence. You can be 99.9% at fault and you still recover for .1%.
6) MA, you can be up to and including 50% at fault and still recover. If P is over 50% at fault, he recovers nothing. “Partial or comparative fault.”

73
Q

When could strict liability potentially apply?

A

1) Domestic Animals
2) Wild Animals
3) Abnormally dangerous activity
4) Products Liability

74
Q

When does strict liability attach to domestic animals? How might this change in MA?

A
Domesticated animals (House pets and farm animals, including honey bees)  
i.	Not strictly liable, BUT you are strictly liable if you have knowledge of the vicious propensity of that animal.  

ii. The vicious propensity must be particular to this animal, not the species.

iii. “The One Bite Rule” – e.g. You are not liable if your dog has never bitten someone, but once the dog bites someone, you are now strictly liable for the next bite(s).
1. Does not apply to trespassers on your land.
2. Does apply to houseguests, pedestrians, dog park patrons, etc.

iv. In MA, you are strictly liable for all dog bites except for trespassers or people who are teasing or tormenting your dog. If P is under age 7, you are strictly liable regardless of whether he is a trespasser or tormentor.

75
Q

When does strict liability attach to wild animals?

A

Wild Animals – Strict liability always. Note - Honey bees kept at a honey farm are domestic animals.

Beware of the testers discussing a bunch of safety precautions. Safety precautions do not matter with SL.

76
Q

When does strict liability attach to abnormally dangerous activities? What are the categories where my “spidey sense” needs to be tingling?

A

Abnormally Dangerous Activities

a. Two-part test
i. Activity creates a foreseeable risk of serious harm even when reasonable care is exercised. That is, can’t be made safe with current technology.
ii. Activity has to be uncommon or rare in the community in which it is being carried out. That is, it is out of context.

b. It will only be on the multi-state and it’ll be one question related to…
i. Explosives; or
ii. Bio-Hazards or Highly Toxic Chemicals; or
iii. Significant amounts of radiation or Nuclear Energy.

c. Adequate precautions do not mean shit.

77
Q

When does strict liability attach to products?

A

3) Strict Liability for Defective Products
a. Make sure we are talking about strict liability and not negligence or warranty.
b. Includes kitchen appliances, landscaping equipment, cars, food, medicine, industrial products, etc. It’s varied.
c. Elements:

i. D must be a merchant (routinely deals in goods of this kind).
1. Casual sellers are not merchants.
2. Service providers that make goods available incidental to the service, but they are not merchants.
3. Commercial Lessors are merchants and can be strictly liable. E.g. a rental car company.
4. Every party in the distribution chain is a merchant and can be sued. No privity of contract. E.g. I buy a saw at Home Depot. I can sue Home Depot and Black and Decker.

ii. Product is defective – They might stipulate that it is defective on the MBE, but they won’t on the essay.
1. Manufacturing – This product is different than all the others off the assembly line and differs in a way that is more dangerous than a consumer would expect. “This one is defective.”

  1. Design – There is a defect, and there is a viable alternative design.
    a. Safer;
    b. Economical (similar price to make); and
    c. Practical (can’t make the product hard to use or interfere with the main goal).
  2. Information/Label
    a. Residual risk that cannot be viably designed out (e.g. a knife but the risk is obvious).
    b. Consumers would not normally be aware of those risks; and
    c. It lacks adequate warning. (Page 6 of instruction booklet in fine print isn’t going to cut it. One language might not be adequate. Words alone might not be adequate.)

iii. Product has not been altered since it left D’s hands.
1. Presumed if it has traveled in ordinary channels of distribution.

iv. P must have been making a foreseeable use at the time of injury.
1. Use can be foreseeable even if it is inappropriate.
2. E.g. the chair crumbles when you stand on it to change a ceiling lightbulb.
3. E.g. the steering wheel comes off your car at 90 m.p.h.

v. Technically speaking, MA does not recognize strict liability for products in tort, but the same outcome results in the UCC’s warranty doctrines. All elements of proof remain the same, it is just labeled differently.

78
Q

What are the three ways to show that a product is defective?

A

ii. Product is defective – They might stipulate that it is defective on the MBE, but they won’t on the essay.
1. Manufacturing – This product is different than all the others off the assembly line and differs in a way that is more dangerous than a consumer would expect. “This one is defective.”

  1. Design – There is a defect, and there is a viable alternative design.
    a. Safer;
    b. Economical (similar price to make); and
    c. Practical (can’t make the product hard to use or interfere with the main goal).
  2. Information/Label
    a. Residual risk that cannot be viably designed out (e.g. a knife but the risk is obvious).
    b. Consumers would not normally be aware of those risks; and
    c. It lacks adequate warning. (Page 6 of instruction booklet in fine print isn’t going to cut it. One language might not be adequate. Words alone might not be adequate.)
79
Q

What defenses are available for strict liability?

A

1) Comparative Responsibility – Works like comparative negligence.
2) P’s stupidity counts against him.
3) E.g. I taunted my neighbor’s tiger.

MA allows the sophisticated user defense in products liability cases so failing to warn of latent defects will not work if the end user knew or should know of the danger.

80
Q

What is nuisance?

A

D has substantially and unreasonably interfered with P’s ability to use and enjoy real estate.

1) E.g.
a. Neighbors that shouldn’t be next to each other (hospital for respiratory infections next to a smoke belching factory.)
b. Loud party house next to quiet neighbors.
c. Bright lights shining into P’s bedroom.

2) Balance each party’s interest in using property as one wants.

3) Is it intolerable?
a. Is it something that happens routinely and you just have to put up with it.
b. E.g. mowing the lawn at 8 a.m. is not intolerable.
c. E.g. church bells are not a nuisance.

4) Does not apply to P’s hypersensitivity.

5) Look for an answer that says something about the law. Key words:
a. “substantial and unreasonable”
b. “balance of interests”

81
Q

When might someone be liable for vicarious liability? Any MA distinctions?

A

Vicarious Liability – Due to a relationship, a third party is liable for someone else’s wrongdoing.

1) Employer-Employee
a. Employer is liable for torts of employee for torts committed within the scope of employment.
i. Intentional Torts are generally outside the scope of employment, UNLESS:
1. If job involves authorization to use force (e.g. a bounce).
2. Job involves friction (e.g. a repo man).
3. Misguided effort to serve the employer’s agenda (e.g. store security guard detains every third shopper without reasonable belief of theft.)

2) Independent contractors are not generally vicariously liable, UNLESS
a. The independent contractor hurts an invitee/customer because it’s non-delegable.
b. Encourages proper vetting in the hiring process.

3) Automobile Owner-Driver
a. Generally, no vicarious liability unless I ask you to run an errand for me.

4) Parent-Child
a. Parents are not liable for the torts of their children.
b. In MA, by statute, parents are liable for intentional torts of children up to $5,000. (Arose from school vandalism.)

NOTE***Vicarious liability is a doctrine of last resort. Always ask, “Can I hold this person liable for his own conduct.”

c. E.g. I lend my car to an obviously drunk person. Liable for negligent entrustment, not vicarious liability.
d. E.g. I do not do due diligence in hiring someone. I am liable for negligent hiring, not vicarious liability.
e. E.g. I have a little kid, and I leave my gun out. My kid shoots someone. This is negligence, not vicariously liable.

82
Q

When may a defendant seek contribution or indemnification?

That is, P sued multiple Ds (A, B, and C). They are jointly liable. A pays P. A goes after B and C for contribution. Under what circumstances will he be successful?

What about indemnification?

MA rule?

A

a. Comparative Contribution (majority view) – Every D is given a % for his degree of fault. Out of pocket D gets back the other parties’ %. If B and C are insolvent, then A is fucked.

b. Indemnification - A gets all his money
i. Vicariously liable parties may be indemnified by the actual tortfeasor.
ii. A retailer may be indemnified by the manufacturer in a strict liability case.

c. MA uses “Equal Fractional Shares” (no shares. 3 Ds = 33% each.)

83
Q

Does a spouse have an action if the other spouse is injured?

A

Yes. Lecturer covered one. Loss of Consortium
1) Uninjured spouse of P has separate claim against D for loss of consortium.

2) Defense against injured spouse (comparative negligence, consent, etc.) may be raised against the spouse.

3) Possible recovery of: (each must be proven).
a. Household services
b. Loss of society/companionship
c. Loss of sex

In MA, dependent children may recover for loss of consortium (minors or adult dependents.)
If parent dies before baby is born, the child may also recover for LoC.

84
Q

What is distinct about damages for a trespass to land in MA?

A

Treble damages for willfully cutting down/carrying away/destroying trees on another’s land.

Single damages if you reasonably thought you owned the land or you were authorized to cut down the trees.

85
Q

Imagine there was a certain person in our law school class with an already poor reputation. Let’s say he said off color things in class, embarrassed himself at internships, said things that made people uncomfortable, kidnapped a classmate, and slashed a classmate’s tires. Then, let’s say I passed out flyers in Boston warning the Commonwealth about this person’s past behavior. Is it possible that MA would consider him “libel proof”?

A

MA does not recognize a difference between libel per se and libel per quod, but it does acknowledge that some people’s reputation is so poor that it is “libel proof”.

86
Q

What is wrongful termination in MA for an at will employee?

A

Generally, if you are an at will employee you may be fired for any reason.

However:

1) Promises in the personnel manual amount to an express contract.
2) The discharge violates public policy (e.g. jury duty or reporting a crime, but not leaving early to pick kids up at daycare.)
3) Breached cov. of good faith and fair dealing (i.e. fired you to deprive of earned salary or commission.)

87
Q

What is the duty to warn in MA?

A

In MA, psychologists and social workers have a duty to warn potential victims of threats made by their patients or to take other reasonable precautions.

88
Q

Is assumption of risk an available defense in MA?

A

No, except in strict liability.

89
Q

In Massachusetts it is illegal for a tavernkeeper to serve, furnish, or supply alcohol to anyone under 21, or to an intoxicated person. In an action in tort against a tavernkeeper, what is the effect of evidence that the tavernkeeper violated this statute?

A

Violation of the statute is evidence of negligence as to a minor. As to an intoxicated person, the plaintiff must show willful, wanton, or reckless conduct.

As to minors, violation of the statute is evidence of negligence and is considered when determining whether a person of ordinary prudence would have served the patron. As to intoxicated persons generally, the plaintiff must show that the tavernkeeper breached a duty to refrain from willful, wanton, or reckless conduct in serving the intoxicated person.

90
Q

The Massachusetts Tort Claims Act (“MTCA”) provides immunity from suit for a wide range of acts and omissions by government actors. In which of the following circumstances would the immunity apply?

A A building is destroyed when the municipal fire department fails to arrive on the scene in time to put out a fire, because the engine was driven negligently.
B A licensing board denies a restaurant’s application for a liquor license because it misplaced part of the application.
C A state prisoner escapes because his cell was left unlocked, and later commits a murder.
D A child is injured when a teeter totter collapses because it was negligently maintained by the city parks department.

A

B A licensing board denies a restaurant’s application for a liquor license because it misplaced part of the application.

The MTCA protects various government actors from suit for certain acts. This includes protection for the negligent acts of licensing boards and the like in actions arising from the granting or withholding of permits or licenses. However, where a fire engine is negligently driven, a prisoner escapes because he was negligently supervised, or public property is negligently maintained, an injured person may recover. Those are essentially ministerial acts that are not immune from liability (as opposed to discretionary acts, which are immune).

91
Q

A duty of care is generally NOT owed to:

A A rescuer, unless the defendant negligently put herself or a third person in peril.
B A viable fetus.
C A third party for whose economic benefit a legal or business transaction is made.
D A discovered trespasser.

A

A A rescuer, unless the defendant negligently put herself or a third person in peril.

92
Q

Which of the following need NOT be shown by the plaintiff under the attractive nuisance doctrine?

A The owner was or should have been aware of the dangerous condition.
B The child was lured onto the property by the attractive nuisance.
C The condition was likely to cause injury because of the child’s inability to appreciate the risk.
D The expense of remedying the situation is slight compared with the magnitude of the risk.

A

B The child was lured onto the property by the attractive nuisance.

The plaintiff does not need to show that the child was lured onto the property by the attractive nuisance. The plaintiff does need to show that the owner was or should have been aware of the dangerous condition, that it was likely to cause injury because of the child’s inability to appreciate the risk, and that the expense of eliminating the danger is slight compared with the magnitude of the risk. Under the attractive nuisance doctrine, a landowner has a duty to exercise ordinary care to avoid reasonably foreseeable risk of harm to children caused by artificial conditions on the property. To recover under this doctrine, the plaintiff must show that (i) there is a dangerous condition present on the land of which the owner is or should be aware, (ii) the owner knows or should know that young persons frequent the vicinity of this dangerous condition, (iii) the condition is likely to cause injury, i.e., is dangerous, because of the child’s inability to appreciate the risk, and (iv) the expense of remedying the situation is slight compared with the magnitude of the risk.

93
Q

If a statute providing for a criminal penalty is applicable to a common law negligence case, the statute’s specific duty will replace the more general common law duty of care. Which of the following does a plaintiff NOT need to show to prove the availability of the statutory standard?

A	The plaintiff is in the class intended to be protected by the statute.
B	The statute was designed to prevent the type of harm that the plaintiff suffered.
C	The plaintiff suffered physical injury because of the defendant’s violation of the statute.
D	The standards set out in the statute are clearly defined.
A

C The plaintiff suffered physical injury because of the defendant’s violation of the statute.

The plaintiff need not suffer physical injury from the defendant’s violation of the statute. While damages is an element of the prima facie case for negligence, any type of damages, including property damages, will suffice. To prove the availability of the statutory standard, a plaintiff must show that the standards set out in the statute are clearly defined. For the statute to apply, (i) the plaintiff must be in the class intended to be protected by the statute, and (ii) the statute must have been designed to prevent the type of injury that he suffered.

94
Q

A bystander who witnesses the defendant negligently injuring another can recover for negligent infliction of emotional distress in most states by showing:

A A close relationship between the bystander and the person injured, the bystander’s presence at the scene of the injury, and the bystander’s observation or perception of the event.
B A close relationship between the bystander and the person injured, the bystander’s presence within the zone of danger from physical injury, and the bystander’s observation or perception of the event.
C The bystander’s presence within the zone of danger from physical injury, and the bystander’s observation or perception of the event.
D A close relationship between the bystander and the person injured, and the bystander’s observation or perception of the event.

A

A A close relationship between the bystander and the person injured, the bystander’s presence at the scene of the injury, and the bystander’s observation or perception of the event.

A bystander must show a close relationship between the bystander and the person injured, the bystander’s presence at the scene of the injury, and the bystander’s observation or perception of the event to recover. Traditionally, a bystander outside the “zone of danger” of physical injury who sees the defendant negligently injuring another could not recover damages for her own distress. A majority of states now allow recovery in these cases as long as (i) the plaintiff and the person injured by the defendant are closely related; (ii) the plaintiff was present at the scene of the injury; and (iii) the plaintiff personally observed or perceived the event. The bystander’s presence within the zone of danger from physical injury is no longer required for a bystander to recover for witnessing an injury to another.

95
Q

A defendant intended to commit an assault on A, but his conduct only constituted a battery on B.
Under the transferred intent doctrine, the defendant is liable for:

A An assault of B.
B An attempted assault of A and a battery of B.
C A battery of B.
D An attempted assault of A and an assault of B.

A

C A battery of B.

(C) The defendant has committed a battery of B when he acts with the intent to commit an assault on A, but his conduct only constitutes a battery on B. The transferred intent doctrine allows an intent to commit a tort against one person to be transferred to the committed tort or to the injured person. It applies to (i) assault, (ii) battery, (iii) false imprisonment, (iv) trespass to land, and (v) trespass to chattels. The defendant is not liable for an assault of B. The committed tort was a battery, and the intent transfers from the assault to the battery. Nor is the defendant liable for an attempted assault of A. There is no tort liability for an attempted assault standing alone. The defendant is liable only because of the transferred intent doctrine, and only to the person harmed. Thus, there is no liability to A here.

96
Q

The “shopkeeper’s privilege” allows a shopkeeper to avoid liability for false imprisonment when detaining a suspect that he reasonably believes has committed a theft.
The shopkeeper also must:

A Conduct the detention in a reasonable manner and detain the suspect for only a reasonable time.
B Conduct the detention in a reasonable manner and notify the police in a reasonable amount of time.
C Detain the suspect for only a reasonable time and notify the police in a reasonable amount of time.
D Conduct the detention in a reasonable manner, detain the suspect for only a reasonable time, and notify the police in a reasonable amount of time.

A

A Conduct the detention in a reasonable manner and detain the suspect for only a reasonable time.

(A) In addition to having a reasonable belief as to the fact of theft, a shopkeeper is required to conduct the detention in a reasonable manner and detain the suspect for a reasonable period of time for the privilege to apply. By statute in some states and case law in others, shopkeepers have been given a privilege to detain someone suspected of shoplifting and thus avoid liability for false imprisonment. The following conditions must be satisfied: (i) there must be a reasonable belief as to the fact of theft; (ii) the detention must be conducted in a reasonable manner and only nondeadly force can be used; and (iii) the detention must be only for a reasonable period of time and only for the purpose of making an investigation. A shopkeeper is not required to notify the police in a reasonable amount of time to avoid liability for false imprisonment when detaining a suspect for shoplifting.

97
Q

Which of the following is NOT part of the prima facie case for intentional infliction of emotional distress?

A Intent to cause the plaintiff to suffer severe emotional distress.
B Evidence of severe emotional distress.
C An act by the defendant amounting to extreme and outrageous conduct.
D Physical symptoms caused by the emotional distress.

A

D Physical symptoms caused by the emotional distress.

(D) Physical symptoms caused by the emotional distress are not required. A prima facie case for intentional infliction of emotional distress requires proof of: (i) an act by the defendant amounting to extreme and outrageous conduct; (ii) intent on the part of the defendant to cause the plaintiff to suffer severe emotional distress, or recklessness as to the effect of the defendant’s conduct; (iii) causation; and (iv) damages—severe emotional distress. Intent to cause the plaintiff to suffer severe emotional distress establishes the intent element of the tort. Reckless disregard of a high probability that emotional distress will result also satisfies the intent element of the tort. Evidence of severe emotional distress must be shown; hurt feelings are not sufficient. An act by the defendant amounting to extreme and outrageous conduct is an element of the tort.

98
Q

Which of the following intentional conduct by the defendant is LEAST likely to constitute a trespass to land?

A Exploding a mine on his own land which causes concussion damage to the plaintiff’s land.
B Throwing a rock onto a plaintiff’s driveway which causes no damage.
C Flooding his own land which causes water damage to the plaintiff’s land from the overflow.
D Chasing someone from his own land onto the plaintiff’s land which causes no damage.

A

(A) If a defendant explodes a mine on his land which causes only concussion damage to a plaintiff’s land, this will likely not constitute a trespass to land. A trespass to land requires a physical invasion of a plaintiff’s real property by a defendant. If no physical object enters onto the plaintiff’s land due to the defendant’s actions, courts will generally not treat the defendant’s conduct as a trespass to land. Instead, this will constitute a case of nuisance or strict liability (if ultrahazardous activities are involved). If a defendant throws a rock onto the plaintiff’s driveway, causes water to flow onto the plaintiff’s land, or chases someone onto the plaintiff’s land, this will likely constitute a trespass to land. Although in none of these scenarios does the defendant actually enter upon the plaintiff’s land, a trespass to land does not require that the defendant personally come onto the land. The tort of trespass to land protects a plaintiff’s exclusive possession of realty from physical invasion, and all that is required to satisfy this element is a physical invasion of the plaintiff’s land. Furthermore, damages are not required for this tort. The defendant has committed a trespass even if the property was not damaged.

99
Q

If the plaintiff establishes res ipsa loquitur, it will have the following effect:

A A directed verdict will not be given for the defendant.
B A directed verdict will be given for the plaintiff.
C The burden of proof is shifted to the defendant.
D A presumption of negligence is created.

A

A A directed verdict will not be given for the defendant.

The circumstantial evidence doctrine of res ipsa loquitur deals with those situations where the fact that a particular injury occurred may itself establish or tend to establish a breach of duty owed. Where res ipsa loquitur has been proven, the plaintiff has made a prima facie case, and a directed verdict will not be given for the defendant. Application of the doctrine does not shift the burden of proof to the defendant, nor does it create a presumption of negligence. Furthermore, the doctrine does not result in a directed verdict for the plaintiff. The defendant may introduce evidence that due care was exercised, and the jury may reject the permissible inference that may be drawn from the res ipsa proof and find for the defendant.

100
Q

There often is more than one cause for an injury. The “but for” test for actual cause applies to:

A Joint causes.
B Alternative causes.
C Superseding causes.
D Concurrent causes.

A

D Concurrent causes.

The “but for” test for actual cause applies to concurrent causes. An act or omission to act is the cause in fact of an injury when the injury would not have occurred but for the act. This test applies in concurrent cause cases, where several acts combine to cause the injury, but none of the acts standing alone would have been sufficient. But for any of the acts, the injury would not have occurred. The “substantial factor” test is used for joint causes, where several causes commingle and bring about an injury, but any one alone would have been sufficient to cause the injury. In that case, it is sufficient if defendant’s conduct was a substantial factor in causing the injury. An alternative causes situation arises when two or more persons have been negligent, but uncertainty exists as to which one caused the plaintiff’s injury. Under this approach, the plaintiff must prove that harm has been caused to him by one of them (with uncertainty as to which one). The burden of proof then shifts to the defendants, and each must show that his negligence is not the actual cause. Superseding causes arise in the context of proximate cause rather than actual cause. In addition to being an actual cause, the defendant’s conduct must also be a proximate cause of the injury. Causes that arise after the defendant’s conduct that contribute to the injury may be so unforeseeable as to be superseding causes, which cut off the defendant’s liability for his original negligent act.

101
Q

Which of the following describes only dependent intervening forces in a proximate cause analysis?

A Efforts to protect person or property and acts of God.
B Subsequent medical malpractice and criminal acts of third persons.
C A subsequent disease and negligence of rescuers.
D A subsequent accident and an intentional tort of a third person.

A

C A subsequent disease and negligence of rescuers.

Dependent intervening forces are normal responses or reactions to the situation created by the defendant’s negligent act. Dependent intervening forces are almost always foreseeable. A subsequent disease is a common dependent intervening force. The original tortfeasor is usually liable for diseases caused in part by the weakened condition in which the defendant has placed the plaintiff by negligently injuring her. Also, negligence of rescuers is a common dependent intervening force. Generally rescuers are viewed as foreseeable intervening forces, so the original tortfeasor usually is liable for their negligence. Efforts to protect person or property are common dependent intervening forces. A defendant is usually liable for negligent efforts on the part of persons to protect the life or property of themselves or third persons endangered by the defendant’s negligence. Subsequent medical malpractice is also a common dependent intervening force. The defendant is usually liable for the aggravation of the plaintiff’s condition caused by the malpractice of the treating physician. A subsequent accident may also be a dependent intervening force if the original injury was a substantial factor in causing the second accident. However, acts of God and intentional torts and criminal acts of third persons are independent intervening forces. Independent intervening forces operate on the situation created by the defendant’s negligence, but they are independent actions rather than natural responses or reactions to the situation. (Note that the defendant may or may not be liable for independent intervening forces. It depends on whether they are foreseeable.)

102
Q

The defendant negligently blocked a road, forcing the plaintiff to take an alternate road that was equally safe. Another driver negligently collided with the plaintiff on that road. The defendant is not liable to the plaintiff because the collision is:

A An unforeseeable result caused by a foreseeable intervening force.
B An unforeseeable result caused by an unforeseeable intervening force.
C A foreseeable result caused by an unforeseeable intervening force.
D An independent result caused by a dependent intervening force.

A

B An unforeseeable result caused by an unforeseeable intervening force.

The defendant is not liable because the collision is an unforeseeable result caused by an unforeseeable intervening force. As a general rule, intervening forces that produce unforeseeable results (i.e., results that were not within the increased risk created by defendant’s negligence) will be deemed to be unforeseeable and superseding. A superseding force is one that serves to break the causal connection between defendant’s initial negligent act and the ultimate injury, and itself becomes a direct, immediate cause of the injury. Thus, the defendant will be relieved of liability for the consequences of his antecedent conduct. Here, the defendant’s negligence was an actual cause of the plaintiff’s injury because it would not have happened but for the defendant’s negligence, but it did not increase the risk that a completely unrelated collision would happen. The result was not caused by a foreseeable intervening force because the other driver’s negligence was completely unrelated to any risk created by the original defendant’s negligence. Similarly, the collision was not a foreseeable result because traveling on the equivalent alternate road did not make a collision foreseeable.The collision is not an independent result caused by a dependent intervening force because dependent forces are those that are normal responses or reactions to the situation created by the defendant’s negligent act, and here the force was unrelated to the defendant’s conduct.

103
Q

Which of the following statements is NOT true under the rule that the tortfeasor takes the victim as he finds him?

A The rule applies to the victim’s existing physical condition but not his mental condition.
B The rule applies in both direct cause cases and indirect cause cases.
C The unforeseeable severity of the plaintiff’s harm does not relieve the defendant of liability.
D The rule is also known as the “eggshell-skull plaintiff” rule.

A

A The rule applies to the victim’s existing physical condition but not his mental condition.

The rule that the tortfeasor takes the victim as he finds him applies to both the victim’s existing physical condition and his mental condition. In both direct cause cases and indirect cause cases, the fact that the extent or severity of the harm was not foreseeable does not relieve defendant of liability; in other words, the unforeseeable severity of the plaintiff’s harm, or its extent, is irrelevant under this rule. This rule is also known as the “eggshell-skull plaintiff” rule.

104
Q

In a negligence action, the plaintiff cannot recover:

A Unforeseeable damages
B Noneconomic damages
C Presumed damages
D Damages for lost future earning capacity

A

C Presumed damages

In a negligence action, the plaintiff cannot recover presumed damages. Damage is an essential element of a plaintiff’s prima facie case for negligence. This means actual harm or injury. Unlike for some intentional torts, damage will not be presumed in negligence. A plaintiff is entitled to all damages that he can prove, even if the extent of the damages was unforeseeable. Permissible damages includes economic damages, such as medical expenses and lost earnings, and
noneconomic damages, such as pain and suffering. The plaintiff is also entitled to damages for lost future earning capacity, discounted to present value to avoid an excess award; i.e., the plaintiff receives an amount that, if securely invested, would produce the income that the jury wishes him to have.

105
Q

For assumption of risk to be available as a defense, the plaintiff must have:

A Ignored the risk and negligently assumed it.
B Known of the risk and voluntarily assumed it.
C Known of the risk and expressly assumed it.
D Ignored the risk and impliedly assumed it.

A

B Known of the risk and voluntarily assumed it.

For assumption of risk to be available as a defense, the plaintiff must have known of the risk and voluntarily assumed it. A plaintiff who ignored a risk or negligently assumed it would be subject to the defense of contributory negligence rather than assumption of risk. For assumption of risk, it is irrelevant that the plaintiff’s choice is unreasonable. While the plaintiff must have known of the risk, it need not be expressly assumed; it may be impliedly assumed. Different rules may apply, but both types of assumption of risk may be raised as a defense.

106
Q

To prove breach of duty in a products liability action based on negligence, the plaintiff must show:

A Res ipsa loquitur.
B The conduct involved was below the level of care generally exercised by the defendant.
C The product was dangerous because it departed from its intended design.
D The defendant supplied a defective product.

A

D The defendant supplied a defective product.

To prove breach of duty in a products liability action, the plaintiff must show (i) negligent conduct by the defendant leading to (ii) the supplying of a defective product by the defendant. Negligent conduct is demonstrated by showing that the defendant’s conduct fell below the standard of care expected of a reasonable person under like circumstances, not the level of care generally exercised by the defendant. To show negligence in a manufacturing defect case, the plaintiff may invoke res ipsa loquitur, but it is not required that the plaintiff prove res ipsa loquitur in establishing breach of duty. A plaintiff may show that a product was dangerous because it departed from its intended design to establish a manufacturing defect, but may instead show that the design itself is deficient (to establish a design defect).

107
Q

In contrast to products liability cases based on negligence, those based on strict liability do not:

A Require an injured bystander to be foreseeable.
B Require that suppliers have an opportunity to inspect.
C Prohibit recovery of solely economic losses.
D Impose liability when an intermediary negligently failed to discover the defect.

A

B Require that suppliers have an opportunity to inspect.

Unlike with products liability cases based on negligence, those based on strict liability do not require that suppliers have an opportunity to inspect. Thus, for a case based on the sale of a defective product, a retailer in a strict liability action may be liable for a manufacturing or design defect simply for being a commercial supplier of that defective product, even if it had no opportunity to inspect the manufacturer’s product before selling it. In a negligence action, the supplier’s negligence must be proved. Products liability cases based on negligence and those based on strict liability both require that an injured bystander be foreseeable. While privity is not required in these cases, and bystanders are protected and may bring a claim under either theory, they must be foreseeable plaintiffs. Liability under these theories applies only to foreseeable plaintiffs. Products liability cases based on negligence and those based on strict liability both prohibit recovery of solely economic losses. The types of damages recoverable under both theories are the same: personal injury and property damages. Economic loss cannot be the sole damage claim. As under claims based on negligence, those based on strict liability will impose liability even though an intermediary negligently failed to discover the defect. The same concepts of proximate cause govern negligence and strict liability actions. The negligent failure of an intermediary to discover a defect is not a superseding cause and does not cut off the supplier’s strict liability. However, if the intermediary’s conduct becomes something more than ordinary foreseeable negligence, then it does become a superseding cause.

108
Q

In a products liability case based in strict liablity, a plaintiff may recover:

A Only damages for economic losses
B Only personal injury damages
C Personal injury damages and property damages
D Personal injury damages, property damages, and damages for economic losses

A

C Personal injury damages and property damages

In a products liability case based on strict liability, a plaintiff may recover both personal injury damages and property damages for the supplying of a defective product. If the plaintiff’s complaint is only that the product does not work as well as expected or requires repairs (i.e., no personal injury or property damages), most courts do not permit recovery of damages for economic losses under either a strict liability or a negligence theory; the plaintiff must bring an action for breach of warranty.

109
Q

How is “actual malice” defined for purposes of the constitutional law of defamation?

A Knowledge that the statement was false, or reckless disregard by the defendant as to the statement’s truth or falsity.
B At least negligence by the defendant as to the statement’s truth or falsity.
C Ill will or spite towards the plaintiff by the defendant.
D Knowledge that the statement will defame the plaintiff, or reckless disregard by the defendant as to the effect of the defamatory language.

A

A Knowledge that the statement was false, or reckless disregard by the defendant as to the statement’s truth or falsity.

“Actual malice,” which must be established by a public official or public figure to recover for defamation, was defined by the Supreme Court as (i) knowledge that the statement was false, or (ii) reckless disregard by the defendant as to the statement’s truth or falsity. Negligence by the defendant as to the statement’s truth or falsity is the minimum standard imposed by the Court on a private figure when the defamation involves a matter of public concern; actual malice is a higher standard. Ill will or spite towards the plaintiff by the defendant is how malice was defined at common law, but is not used by the Supreme Court to define malice. Knowledge that the statement will defame the plaintiff or reckless disregard by the defendant as to the effect of the defamatory language is not how malice was defined by the Supreme Court. The Court used actual malice to describe the defendant’s necessary state of mind as to the statement’s truth or falsity, not its defamatory nature.

110
Q

Which one of the following types of common law defamation requires pleading and proof of special damages?

A Libel not falling into one of the four per se categories.
B Slander not falling into one of the four per se categories.
C Slander not defamatory on its face.
D Libel not defamatory on its face.

A

(B) Slander not falling into one of the four per se categories requires pleading and proof of special damages (i.e., pecuniary losses). Injury to reputation is not presumed for spoken defamation unless it falls into one of the four categories of slander per se. Damages will be presumed if the defamation (i) disparages the plaintiff in the conduct of her business or occupation; (ii) asserts that the plaintiff is suffering from a loathsome and communicable disease; (iii) alleges that the plaintiff has committed a serious crime or crime of moral turpitude; or (iv) imputes unchastity to a female plaintiff. Libel does not require special damages, even libel not falling into one of the four per se categories. In most jurisdictions, general damages are presumed by law for all libel; i.e., special damages need not be established. The fact that slander is not defamatory on its face does not affect whether special damages are required; rather, whether the slander falls into the per se categories determines that requirement. Only a minority of courts require special damages for libel not defamatory on its face. As stated above, most states do not require special damages for libel.

111
Q

Which of the following invasion of privacy branches require the plaintiff to show “publicity”?

A Intrusion on plaintiff’s affairs or seclusion and public disclosure of private facts about plaintiff.
B Intrusion on plaintiff’s affairs or seclusion and publication of facts placing plaintiff in a false light.
C Publication of facts placing plaintiff in a false light and public disclosure of private facts about plaintiff.
D Intrusion on plaintiff’s affairs or seclusion, publication of facts placing plaintiff in a false light, and public disclosure of private facts about plaintiff.

A

C Publication of facts placing plaintiff in a false light and public disclosure of private facts about plaintiff.

The invasion of privacy branches based on publication of facts placing the plaintiff in a false light and public disclosure of private facts about the plaintiff require “publicity” concerning the false light or private facts—i.e., widespread dissemination of the facts. Mere publication to a third person is not sufficient for liability. In contrast, invasion of privacy based on intrusion on the plaintiff’s affairs or seclusion requires neither publication nor publicity – just the act of intruding.

112
Q

What must be shown for a qualified privilege to apply for defamatory statements?

A There must be a common interest between the publisher and the recipient
B The statement must be reasonably relevant to the interest being protected
C The statement must not have been made with ill will
D The statement must be in response to a request by the recipient

A

B The statement must be reasonably relevant to the interest being protected

For a qualified privilege to apply, the statement must be reasonably relevant to the interest being protected. The privilege does not encompass the publication of irrelevant defamatory matter unconnected with the public or private interest entitled to protection. However, there need not be a common interest between the publisher and the recipient; statements made in the interest of one party but not the other may fall within a qualified privilege. While a qualified privilege does not apply if the defendant acted with actual malice, that term refers to knowledge of falsity or reckless disregard of truth rather than ill will; hence, a defendant acting with ill will may still assert a qualified privilege. Finally, the statement need not be in response to a request by the recipient; if the publisher has a relationship with the recipient, volunteered statements may fall within the privilege.

113
Q

Which of the following best states who may bring a strict liability action against a defendant engaging in abnormally dangerous activities?

A Anyone injured as a result of the dangerous propensity of the activity
B Any foreseeable plaintiff injured as a result of the dangerous propensity of the activity
C Anyone directly injured by the activity
D Any foreseeable plaintiff as long as she was directly injured by the activity

A

B Any foreseeable plaintiff injured as a result of the dangerous propensity of the activity

A defendant engaging in an abnormally dangerous activity may be liable only to foreseeable plaintiffs injured as a result of the dangerous propensity of the activity. This is the best statement of the scope of the duty owed. In most states, a defendant will be liable only to those persons to whom a reasonable person would have foreseen a risk of harm under the circumstances. In general, strict liability is not imposed for injuries to a plaintiff to whom no reasonable person would have foreseen a danger. The defendant will not be strictly liable to all plaintiffs who were directly injured by the activity. Rather, the harm must result from the kind of danger to be anticipated from the abnormally dangerous activity; i.e., it must flow from the “normally dangerous propensity” of the activity involved. Conversely, a foreseeable plaintiff may recover even if she was not directly injured by the activity as long as the injury was from the dangerous propensity (e.g., injuries caused by fleeing the danger from the activity).

114
Q

A principal will be vicariously liable for the tortious acts of her independent contractor:

A If the independent contractor is engaged in inherently dangerous activities.
B Under the doctrine of respondeat superior.
C If the principal negligently selected the independent contractor.
D If the principal negligently supervised the independent contractor.

A

A If the independent contractor is engaged in inherently dangerous activities.

A principal will be vicariously liable for the tortious acts of her independent contractor if the independent contractor is engaged in inherently dangerous activities. In general, a principal will not be vicariously liable for tortious acts of an independent contractor. Two broad exceptions exist, however: (i) the independent contractor is engaged in inherently dangerous activities, e.g., excavating next to a public sidewalk, blasting; or (ii) the duty, because of public policy considerations, is simply nondelegable, e.g., the duty of a business to keep its premises safe for customers. Respondeat superior is the doctrine that makes employers vicariously liable for the torts of employees; it does not apply to independent contractors. A principal may be liable for negligently selecting or supervising an independent contractor. However, that liability is for her own negligence; it is not vicarious liability.

115
Q

The right of contribution among tortfeasors:

A Imposes contribution based on equal shares of the overall liability.
B Provides for apportionment of damages in the absence of joint and several liability.
C Does not apply against a tortfeasor who is immune from liability.
D Applies to intentional torts.

A

C Does not apply against a tortfeasor who is immune from liability.

The right of contribution among tortfeasors is a device whereby responsibility is apportioned among those who are at fault. However, it does not apply against a tortfeasor who is immune from liability. If the contributing tortfeasor has a defense that would bar liability, such as intra-family tort immunity, she is not liable for contribution. In most states, contribution is based on relative fault of the various tortfeasors rather than on equal shares of the overall liability. Contribution does not provide for apportionment of damages in the absence of joint and several liability; rather, it can only operate in response to joint and several liability, because it allows any tortfeasor required to pay more than his share of damages under joint and several liability rules to have a claim against the other jointly liable parties for the excess. Contribution does not apply to intentional torts in most states.

116
Q

For purposes of private nuisance, the interference with the plaintiff’s use of the land is unreasonable only if:

A The interference is offensive or annoying to an average person in the community.
B The remedy of damages is unavailable or inadequate.
C The nuisance is a “nuisance per se.”
D The severity of the plaintiff’s inflicted injury outweighs the utility of the defendant’s conduct.

A

D The severity of the plaintiff’s inflicted injury outweighs the utility of the defendant’s conduct.

The interference with the plaintiff’s use of the land will be considered unreasonable under nuisance law when the severity of the plaintiff’s inflicted injury outweighs the utility of the defendant’s conduct. In balancing these respective interests, courts take into account that every person is entitled to use his own land in a reasonable way, considering the neighborhood, land values, and existence of any alternative courses of conduct open to the defendant. Whether the interference is offensive or annoying to an average person in the community is the test for whether the interference is substantial, which is a separate requirement for establishing a nuisance. Whether the remedy of damages is unavailable or inadequate determines only whether the plaintiff may be able to obtain an injunction remedy. A nuisance is sometimes called a “nuisance per se” when it is based on strict liability (e.g., a nuisance arising from an abnormally dangerous activity).

117
Q

A small cruise ship struck a whale swimming underwater, causing the ship to suddenly lurch sideways. A passenger on the ship who was walking down a corridor lost his balance and bumped his head on the edge of a doorway. Because of a previously existing medical condition that made him susceptible to bleeding on the brain, he suffered a cerebral hemorrhage and permanent mental impairment, despite prompt medical attention on the ship.

The passenger brought suit against the cruise ship owner for his damages. At trial, the passenger presented evidence of how he was injured as he walked down the hallway, his previous medical condition, and his medical expenses and other damages. The cruise ship owner presented evidence that the cruise ship was following its approved route and that the whale could not have been detected before impact, and that the bump would not have injured someone in ordinary health. At the close of the evidence, the cruise ship owner moved for a directed verdict.

How should the court rule?

A Grant the motion, because there is no evidence that the crew operated the ship negligently.
B Grant the motion, because the cruise ship owner introduced uncontroverted evidence that a person in normal health would not have been injured by the bump.
C Deny the motion, because the jury could find that the cruise ship owner, as a common carrier and innkeeper, breached its high duty of care to the passenger.
D Deny the motion, because the fact that the severity of the passenger’s injuries was not
foreseeable does not cut off the cruise ship owner’s liability.

A

A Grant the motion, because there is no evidence that the crew operated the ship negligently.

The court should grant the cruise ship owner’s motion because the passenger has not established a prima facie case of negligence against the cruise ship. To establish a prima facie case for negligence, a plaintiff must show (i) a duty of care, (ii) breach of that duty, (iii) actual and proximate cause, and (iv) damages. As a common carrier and/or an innkeeper, the cruise ship owed its passengers a high duty of care, and therefore would be liable for slight negligence. However, the passenger has offered no evidence to establish that the cruise ship employees breached that duty, and res ipsa loquitur is not applicable here because the collision with the whale swimming underwater is not the type of event that would occur only as a result of negligence. Because the passenger failed to establish breach of duty, the court should grant the cruise ship owner a directed verdict. (B) is incorrect because the cruise ship owner does not need that evidence to prevail. While evidence that a person in normal health would not have been injured by the bump supports the cruise ship’s other evidence that it exercised due care, it is not necessary because the passenger has failed to offer evidence that the cruise ship owner breached its duty. On the other hand, if the cruise ship owner had breached its duty of care to its passengers, the fact that a person in normal health would not have been injured by the bump on the head would not be a defense to liability. If a defendant’s negligence causes an aggravation of a plaintiff’s existing physical illness, the defendant is liable for the damages caused by the aggravation. (C) is incorrect because, as discussed above, the passenger has failed to present evidence that the cruise ship owner breached the high duty of care that it owed to its guests. (D) is incorrect even though it is a true statement of law, as discussed above. The reason the cruise ship owner prevails is because the passenger has failed to establish a prima facie case.

118
Q

A patient troubled by an irritating skin rash consulted a dermatologist for treatment. The dermatologist diagnosed the rash as a genetic condition that had no cure and would ultimately spread and lead to disfigurement. The patient was shocked and distressed by the diagnosis. On the advice of her family, a week later the patient consulted another doctor. That doctor immediately diagnosed the skin rash as a common bacterial infection and prescribed an ointment that cleared up the condition in a few days. Because the doctor was a friend of the family, the patient was not charged for that visit.

Can the patient recover from the dermatologist for the emotional distress caused by his erroneous diagnosis?

A Yes, but only if the patient’s distress caused her some physical injury.
B Yes, because the misdiagnosis by the dermatologist caused the patient actual harm.
C No, because the patient did not have to pay for the second doctor visit.
D No, because the dermatologist’s conduct did not create a foreseeable risk of physical injury to the patient.

A

B Yes, because the misdiagnosis by the dermatologist caused the patient actual harm.

The patient’s distress is a recoverable element of damages caused by the dermatologist’s breach of duty to her. A medical specialist such as the dermatologist owes a duty to possess and exercise the degree of knowledge and skill that dermatologists across the nation exercise. He breached that duty by misdiagnosing a common skin infection that another doctor was able to diagnose immediately. His failure to properly diagnose the condition was the actual and proximate cause of injury to the patient; but for the misdiagnosis, she would not have had to continue suffering from the rash until the other doctor properly treated it. The continuation of the rash and any pain and suffering from it are compensable damages that she can recover from the dermatologist. Also compensable is the emotional distress that she suffered because of the misdiagnosis. While recovery for negligent infliction of emotional distress is not always available in many jurisdictions when there is no other injury caused by the breach, these restrictions do not apply when plaintiff is the victim of another tort that causes physical injury. Plaintiff can recover damages for emotional distress that arise from the tortious conduct. Hence, (A) is incorrect. (C) is incorrect because the patient has suffered compensable injury regardless of whether she had to pay for the second doctor visit. The continuation of the skin rash until she saw the other doctor suffices as the damage element of the prima facie case. (D) is incorrect because, given the patient’s physical condition, a failure to make a proper diagnosis did create a foreseeable risk that she would continue to suffer from a painful condition that could otherwise have been alleviated. Thus, the dermatologist’s conduct constituted a breach of duty.

119
Q

A motorcyclist was injured in a collision and suffered $100,000 worth of injuries, including $20,000 in hospital and physician’s bills. The motorcyclist’s medical insurance company paid her $20,000 to cover hospital and medical expenses. Later, she filed suit against the driver of the car that struck her motorcycle. When the case came to trial, the jury agreed with the motorcyclist’s contention that her injuries were worth $100,000. The jury also determined that the motorcyclist was 30% negligent and that the driver was 70% negligent.

How much should the motorcyclist recover from the driver?

A $100,000.
B $70,000.
C $56,000.
D $50,000.

A

B $70,000.

The motorcyclist should recover $70,000 from the driver. Under a pure comparative negligence system, a contributorily negligent plaintiff is allowed to recover a percentage of her damages. The plaintiff’s damages are reduced according to her proportionate share of the fault. Thus, the motorcyclist can recover 70% of her total of $100,000 in damages because she was 30% at fault, leaving her with a recovery of $70,000. (A) is incorrect because it fails to reflect the reduction in damages required under comparative negligence. Because the motorcyclist was 30% negligent, she cannot recover the entire $100,000. (C) is incorrect because it is derived from an initial reduction of damages by the amount of the insurance payments ($100,000 minus $20,000, leaving $80,000). This $80,000 figure is then reduced by the 30% negligence of the motorcyclist, leaving an amount of $56,000. However, as a general rule, damages are not reduced or mitigated by reason of benefits received by the plaintiff from other sources, such as health insurance. Thus, the 30% reduction is made from the figure of $100,000, not from $80,000. Similarly, (D) is incorrect because it is derived from a reduction of the $70,000 proportionate recovery by the $20,000 insurance payment. As noted above, the insurance payments are not allowed to reduce damages. Therefore, the $20,000 paid by the motorcyclist’s insurance company will not reduce the $70,000 in damages to which she is entitled.

120
Q

A man working at a clothing store discovered that his girlfriend, a coworker, had been taking money from the cash register. Not wanting to be a party to the situation, he ended the relationship and found another job. Not long after this, the man’s new boss, who knew why the man had quit, came into the clothing store. He asked the girlfriend if she missed her boyfriend working with her at the store. She replied, “Yes, but when we found that he was stealing from the cash register, we had no choice but to let him go.”

If the man sues his former girlfriend for defamation, the fact that the new boss knew the truth of why the man had left his job at the store will have what result?

A It will act as a complete defense to an action for defamation.
B It will establish that the man has not suffered any actual injury.
C It may diminish the damages that the man would be entitled to recover.
D It proves that the girlfriend had no reasonable ground for believing that the man was fired for dishonesty.

A

C It may diminish the damages that the man would be entitled to recover.

The new boss’s knowledge of the true circumstances behind the man’s departure from the store may diminish the man’s recovery. The girlfriend is liable for defamation because she made a defamatory statement about the man to a third person. As long as it is understood in its defamatory sense, an accusation need not be believed to be actionable. Because the statement that he was stealing at his job constituted slander per se, damages are presumed. Hence, (A) is wrong. (B) is wrong because actual injury encompasses not only damage to reputation but also humiliation and mental distress, for which the man could recover even though the new boss did not believe the girlfriend’s statement. (D) is wrong because the fact that the new boss did not believe the statement does not prove lack of basis for the girlfriend to have made it.

121
Q

A columnist for a major metropolitan newspaper had a very antagonistic relationship with the city’s mayor. When a restaurant owned by the columnist’s family was shut down by city health inspectors, the columnist responded with a column publicizing the shutdown and asserting that it was in retaliation for his prior columns in which he had criticized the mayor. In fact, the mayor had nothing to do with the action by the city health inspectors. While the columnist had no evidence of the mayor’s involvement, he believed that there was a connection because “that’s how the city works.”

Can the mayor recover against the columnist for defamation?

A No, because the columnist did not act with actual malice.
B No, because the columnist had a qualified privilege to explain why he believed his family’s business was shut down.
C Yes, because the columnist’s hostility toward the mayor establishes malice so as to overcome any qualified privilege the columnist had.
D Yes, because the columnist should have investigated the accuracy of his claims before publishing the column.

A

A No, because the columnist did not act with actual malice.

The mayor cannot recover against the columnist because he did not act with actual malice. A public official, such as a mayor, may not recover for defamatory words relating to his official conduct unless there is clear and convincing proof that the statement was made with actual malice, which is defined as knowledge that the statement was false or reckless disregard as to truth or falsity. Reckless conduct is not measured by whether a reasonable person would have investigated before publishing; rather, there must be a showing that the defendant in fact (subjectively) entertained serious doubts as to the truthfulness of his publication. Here, while the columnist had no evidence of the mayor’s involvement with the action of the health inspectors, he believed that there was a connection based on his belief as to how the city operates. Hence, he has not acted with actual malice and is not liable to the mayor for defamation. (B) is incorrect because the columnist’s qualified privilege applies only to statements made to defend his own actions, property, or reputation. Even if it were to apply to his explanation of why his family’s restaurant was shut down, his statements in the column were beyond the scope of the privilege, which does not extend to making a statement to a mass audience whose reading of the statement would not reasonably further his interest in defending himself. Here, the publication in his newspaper column of his explanation as to why the restaurant was shut down was beyond the scope of any privilege he may have had. (C) is incorrect because malice that will result in the loss of a qualified privilege is defined by most courts as knowledge of falsity or reckless disregard as to truth or falsity, rather than hostility or ill-will. As long as the defendant is using a proper occasion for a qualified privilege in a proper way, he will not lose this privilege simply because he bears ill-will toward the plaintiff. (D) is incorrect because the fact that the columnist should have investigated the accuracy of his assertions and did not only establishes negligence on his part. As discussed above, the mayor, as a public official, must show at least reckless disregard as to truth or falsity to recover in a defamation action.

122
Q

A golfer and her instructor were playing golf in a foursome when the golfer became very annoyed with critical comments made by the instructor. To show the other golfers in the group how annoyed she was with her instructor, the golfer stood a few yards behind him while the instructor was teeing off and swung a club at him. The instructor, who was focusing on his shot, was not within range of the club but unfortunately the club slipped out of the golfer’s hands and struck the instructor in the head, injuring him.

If the instructor brings a battery action against the golfer, will he recover?

A Yes, because the golfer acted intentionally and caused harmful contact to her instructor.
B Yes, because the golfer intended to cause the instructor reasonable apprehension of imminent harmful contact.
C No, because the golfer did not intend to cause harmful or offensive contact.
D No, unless the golfer acted unreasonably in swinging the club at her instructor.

A

C No, because the golfer did not intend to cause harmful or offensive contact.

The golfer will not be liable because she did not intend to cause harmful or offensive contact. The prima facie case for battery has the following elements: (i) an act by the defendant that brings about harmful or offensive contact to the plaintiff’s person; (ii) intent on the part of the defendant to bring about harmful or offensive contact to the plaintiff’s person; and (iii) causation. Here, the golfer did not have the intent to cause harmful or offensive contact. Hence, she will not be guilty of battery. (A) is incorrect because even though the golfer had the intent to swing the club, she did not have the intent required for battery—to cause harmful or offensive contact to another. (B) is incorrect because the facts do not support an intent to cause an assault. Under the transferred intent doctrine, an intent to cause an assault (intent to cause apprehension of imminent harmful or offensive contact) will satisfy the intent requirement for battery when the other elements of battery are present. Here, however, the golfer was standing behind the instructor and was intending only to show the other golfers how annoyed she was. No intent to commit assault is apparent here. (D) is incorrect because it describes a negligence standard. The instructor may be able to recover against the golfer in a negligence cause of action if the golfer acted unreasonably in swinging the club, but this does not establish intent for a battery action.

123
Q

A new homeowner had two dogs that frequently barked at birds and squirrels in the yard, especially during the day while the homeowner was at work. A neighbor who worked nights was aggravated by the barking, which disturbed his sleep, and decided to let the homeowner know how he felt. One evening, upon learning that the homeowner was entertaining her boss and several clients, the neighbor came to her front door with a boombox and started playing a recording of the dogs barking, putting it at full volume. When the homeowner came to the door, he began yelling at her and berating her in front of her guests for having no consideration for her neighbors, while continuing to play the recording. The homeowner was very upset, especially because her guests decided that they had better leave, and she ended up losing a bonus that her boss was going to give her at the end of the evening.

If the homeowner asserts a claim based on intentional infliction of emotional distress against the neighbor, what will be the probable result?

A The homeowner will prevail because the neighbor’s conduct was extreme and outrageous.
B The homeowner will prevail because she suffered pecuniary harm from the neighbor’s conduct.
C The neighbor will prevail because the homeowner suffered no physical harm.
D The neighbor will prevail if the barking from the homeowner’s dogs is judged to constitute a nuisance.

A

A The homeowner will prevail because the neighbor’s conduct was extreme and outrageous.

The homeowner will probably prevail on a claim for intentional infliction of emotional distress because the neighbor’s conduct was sufficiently extreme and outrageous and the other elements of the tort are present. Intentional infliction of emotional distress requires: (i) an act by defendant amounting to extreme and outrageous conduct; (ii) intent to cause severe emotional distress or recklessness as to the effect of defendant’s conduct; (iii) causation; and (iv) damages. “Outrageous conduct” is extreme conduct that transcends all bounds of decency. The neighbor’s use of the recording and his insults against the homeowner for the benefit of her guests would probably qualify as extreme and outrageous conduct, particularly because there is no evidence that he had previously tried to resolve the problem with the homeowner in a more civilized manner. The neighbor had the requisite intent (either he intended to cause emotional distress or he was reckless as to its effect), there was causation, and the homeowner suffered damages (i.e., she was severely distressed) as a result of the neighbor’s actions. (B) is wrong because pecuniary harm is not required for purposes of this tort—all that is required is severe emotional distress. (C) is wrong because, in contrast to negligent infliction of distress, intentional infliction of distress does not require proof of physical harm to recover. (D) is wrong because the fact that the barking constituted a nuisance would not be a defense to conduct amounting to intentional infliction of distress; abatement of a private nuisance by self-help must be preceded by notice to the other party and must be conducted in a reasonable manner.

124
Q

A college student borrowed his roommate’s notebook computer without permission because he needed to write a term paper that was due the next day. While the computer was sitting open on the student’s desk overnight, a water pipe in the ceiling began leaking and water dripped down on the computer, rendering it inoperable. A computer repair service estimated that it would cost $500 to repair all the damaged components. At the time it was damaged, the computer was worth $700.

If the roommate sues the student for the damage caused to the computer, what will be the extent of his recovery?

A Nothing, because the damage occurred through no fault of the student.
B Loss of use damages for the time it was in the student’s possession.
C $500 in damages.
D $700 in damages.

A

D $700 in damages.

The roommate can recover $700 in damages from the student for conversion. To establish a prima facie case of conversion, the following elements must be proved: (i) an act by the defendant interfering with the plaintiff’s right of possession in the chattel that is serious enough in nature or consequence to warrant that the defendant pay the full value of the chattel; (ii) intent to perform the act bringing about the interference with the plaintiff’s right of possession, and (iii) causation. Even if the conduct is wholly innocent, liability may attach where the interference is serious in nature. Accordingly, accidentally causing damage to another’s chattel may constitute a conversion when the damage occurred while the defendant was using the chattel without permission. Here, the student interfered with the roommate’s right of possession in the computer by taking it without permission, and it sustained damages of over 70% of its value while in the student’s possession. Hence, the student has committed a conversion. The plaintiff in a conversion case is entitled to damages for the fair market value of the chattel at the time and place of the conversion, which in this case was $700. (A) is incorrect because even though the student was not at fault in the water pipe leaking, the damage occurred while the computer was wrongfully in his possession. (B) is incorrect. Had the computer not been damaged, the roommate’s recovery would be limited to loss of use damages under a trespass to chattels theory. However, the serious damage that occurred while the computer was in the wrongful possession of the student warrants a recovery for conversion. (C) is incorrect because the damages remedy for conversion is the fair market value; in effect, there is a forced sale of the item. The student may keep the computer but he is liable to the roommate for the entire value of the computer rather than just the cost of repairs.

125
Q

A trainer of homing pigeons brought several of them to a park that he often used for training. He had trained this group of pigeons carefully and was confident that they would readily find their way home. When they were released, one of the pigeons inexplicably turned in the opposite direction from home. Several blocks away at the other end of the park, it collided with a radio-controlled model airplane that its owner had just purchased and was trying out for the first time. The collision sent the airplane out of control; it dipped low across a highway and was struck and run over by a truck. The airplane owner sued the pigeon trainer for the destruction of his airplane. The parties stipulated to the above facts and the airplane owner presented evidence of his damages. The trainer then moved for a directed verdict.Should it be granted?

A No, because the trainer’s pigeon caused the destruction of the airplane.
B No, because the jury could find negligence on the trainer’s part under the doctrine of res ipsa loquitur.
C Yes, because the truck, rather than the pigeon, was the direct cause of the airplane’s destruction.
D Yes, because the trainer took reasonable care in training his pigeons.

A

D Yes, because the trainer took reasonable care in training his pigeons.

The court should grant a directed verdict for the trainer because the airplane owner has not shown that the trainer breached any duty that he owed to him. A prima facie case of negligence requires plaintiff to show the following elements: (i) the existence of a duty on the part of the defendant to conform to a specific standard of conduct for the protection of the plaintiff against unreasonable risk of injury, (ii) breach of that duty by the defendant, (iii) that the breach of duty was the actual and proximate cause of plaintiff’s injury, and (iv) damage to plaintiff’s person or property. Here, it is doubtful that the trainer’s releasing his pigeons created any duty to other users of the park. To the extent that it did, the fact that he had taken great care to train them to return directly to their roosts indicates that he did not breach his duty to the airplane owner. Because the airplane owner has offered no other evidence of negligence, nor any reason to impose strict liability on the trainer (as discussed below), the trainer’s motion for a directed verdict should be granted. (A) is incorrect because that choice indicates the imposition of a strict liability standard on the trainer. The owner of a domestic or inherently nondangerous animal is not strictly liable for the injuries it causes. The conduct of the trainer’s homing pigeon would not make the trainer liable in the absence of some negligence on his part. (B) is incorrect because the doctrine of res ipsa loquitur applies only to situations where the fact that a particular injury occurred itself establishes that defendant breached a duty. If the doctrine is applicable, no directed verdict may be given for defendant because plaintiff has established a prima facie case. However, the accident must be the type that would not normally occur unless someone was negligent. The collision between the trainer’s homing pigeon and the model airplane is not that type of accident; by itself, it provides no suggestion that anyone was negligent. (C) is incorrect because the truck is not a superseding force that breaks “the causal connection” between the action of the trainer’s pigeon and the airplane’s destruction. In indirect cause cases, where a force came into motion after defendant’s act and combined with it to cause injury to plaintiff, defendant will still be potentially liable for foreseeable intervening forces that are within the increased risk caused by his acts. Even if the intervening force is independent (i.e., not a natural response or reaction to the situation), it will be foreseeable where defendant’s negligence increased the risk that the independent force would cause harm. Hence, if the trainer were negligent in releasing his pigeon, the fact that the destruction of the airplane was directly caused by the truck would not relieve the trainer from liability, because the initial collision with the pigeon caused the airplane to go out of control and created a substantial risk that it would be damaged by an intervening force.

126
Q

A bicyclist was riding his bicycle in the street when a negligently driven car struck the bike, knocking the bicyclist off the bike and breaking his right ankle. The driver of the car immediately stopped and went to his assistance. She got him to his feet and was slowly moving him toward the curb when a negligently driven taxicab struck him in the left leg. The bicyclist required surgery on both his right ankle and his left leg.If the bicyclist sues the driver and the cabbie, which of the following best states his right to recover?

A He can recover from either the driver or the cabbie for all of his injuries because the driver and the cabbie are jointly and severally liable.
B He can recover from the driver only for the injury to his right ankle and recover from the cabbie only for the injury to his left leg.
C He can recover from either the driver or the cabbie for the injury to his left leg and recover from the driver only for the injury to his right ankle.
D He cannot recover against the driver for the injury to his left leg unless the jury determines that the driver acted negligently when she came to his aid.

A

C He can recover from either the driver or the cabbie for the injury to his left leg and recover from the driver only for the injury to his right ankle.

The bicyclist can recover from either party for the left leg injury but only from the driver for the right ankle injury. When two or more tortious acts combine to proximately cause an indivisible injury to a plaintiff, each tortfeasor is jointly and severally liable to the plaintiff for the entire damage incurred. Joint and several liability applies even though each tortfeasor acted entirely independently. However, if the actions are independent, plaintiff’s injury is divisible, and it is possible to identify the portion of injuries caused by each defendant, then each will be liable only for the identifiable portion. Here, the cabbie would not be liable for the injury to the right ankle, because the cabbie did not cause the injury. (A) is therefore incorrect. With regard to the left leg, the cabbie was not the only cause of that injury. The original tortfeasor is liable for harm caused by the negligence of third persons when such negligence was a foreseeable risk created by the original tortfeasor’s conduct. Here, as a result of the driver’s original negligence, the bicyclist was in a position of danger while he was still in the street. The negligence of the cabbie in striking the bicyclist was a foreseeable risk while the bicyclist was in the street; it is therefore a foreseeable intervening force that will not cut off the driver’s liability. Hence, both the driver and the cabbie will be jointly and severally liable for that injury. (B) is therefore incorrect. (D) is incorrect because the driver remains responsible for the foreseeable consequences of her original negligence in striking the bicyclist, regardless of whether she acted with due care when she came to his aid.

127
Q

A housecleaning agency was given a key to a customer’s house so that the agency could have its employees clean while the homeowner was away. After a maid sent by the agency had finished and left the homeowner’s house, she went back because she had forgotten her cigarettes. She neglected to lock the door when she left the second time because she was already late for the next job. When the homeowner returned after a few days away, she discovered that her house had been ransacked and several items of jewelry stolen. The front door was open, and there were no signs of forced entry.If the homeowner brings an action against the agency that employed the maid, what is the likely result?

A She will not prevail, because she is limited to claims for breach of contract based on her agreement with the agency.
B She will not prevail, because the act of the burglar was an independent superseding cause of the homeowner’s loss.
C She will prevail, because the maid’s failure to lock the door created the risk that someone might enter and take the homeowner’s valuables.
D She will prevail, because when the maid returned after having completed her work, she was technically a trespasser, making the agency vicariously liable for any damage she caused to the premises.

A

C She will prevail, because the maid’s failure to lock the door created the risk that someone might enter and take the homeowner’s valuables.

The homeowner will prevail because the maid’s negligence increased the risk of criminal conduct by a third party. Criminal acts and intentional torts of third persons are foreseeable independent intervening forces if the defendant’s negligence created a foreseeable risk that they would occur. Here, the maid’s failure to lock the door was negligent because it created a risk of burglary; hence, the burglary does not cut off the agency’s liability for the maid’s negligence. As the maid’s employer, the agency is vicariously liable under respondeat superior. (A) is wrong because there is nothing in the facts to indicate that the homeowner waived her right to bring tort claims against the agency; having a contractual relationship with a party does not automatically preclude bringing a tort action against the party. (B) is wrong because the burglary was not a superseding cause of the loss; it was within the increased risk caused by the maid’s negligence. (D) is wrong because she reentered to retrieve a personal item that she had brought with her when she went to the job; her return just to get the item was within the scope of her employment and would not make her a trespasser.

128
Q

A homeowner bought “20-pound test” fishing line for hanging potted plants on his porch. “20-pound test” in the fishing industry means that fishing line will not break under an initial stress of up to 20 pounds when a hooked fish tugs against the line, but not that it will support a constant 20-pound weight. Most sportfishers are aware of this technical meaning, but most laypersons are not, and the manufacturer put no warnings or explanations on the package in which the line was sold. The homeowner hung a 15-pound basket from his front porch, directly above an old-fashioned porch swing. A friend visiting the homeowner was sitting on the swing when the line holding the basket broke, causing the plant to fall and strike the guest on the head.In a jurisdiction following the traditional rules for landowners and possessors of land, will the guest prevail against the homeowner in a suit to recover damages for her injuries?

A Yes, because she was a social guest.
B Yes, because the homeowner was negligent in hanging the plant.
C No, because the homeowner could not be expected to know the technical meaning of “20-pound test.”
D No, because she was not a foreseeable plaintiff.

A

C No, because the homeowner could not be expected to know the technical meaning of “20-pound test.”

The guest will not prevail in a suit against the homeowner. As a social guest of the homeowner’s, the guest is deemed to be a licensee; i.e., one who enters onto land with the owner’s permission for her own purpose or business rather than for the owner’s benefit. In a jurisdiction following the traditional rules for landowners and possessors of land, the owner owes a licensee the duty to warn of or make safe a dangerous condition known to the owner that creates an unreasonable risk of harm to the licensee and that the licensee is unlikely to discover. The owner has no duty to a licensee to inspect for defects nor to repair known defects. The homeowner, as a person who was not involved with fishing, had no reason to suspect that a fishing line that was “20-pound test” could not support the constant weight of a 15-pound basket. Thus, the homeowner did not know of the dangerous condition present in the form of the basket overhanging his porch. Because the homeowner was unaware of the danger, he was under no duty to warn the guest, a licensee, of the dangerous condition. Having violated no duty owed to the guest, the homeowner will not be held liable for her injuries. (A) is accurate in stating that the guest was a social guest. However, as detailed above, the duty owed to a guest is simply to warn of concealed dangerous conditions of which the owner is aware. The homeowner had no duty to warn of a danger of which he neither knew nor had reason to know. (B) is incorrect because there is no indication either that the homeowner hung the basket in a negligent manner or that he was negligent in failing either to warn the guest or to be aware of the danger. The homeowner appears to have acted as would a reasonable person with no knowledge of the meaning of technical terms of fishing. (D) is incorrect because a social guest would indeed be a foreseeable plaintiff. If the homeowner had been negligent in hanging the basket directly above the swing, it would have been reasonably foreseeable that an injury would befall any person who sat on the swing. Thus, (D) reaches the correct result that the guest will not prevail, but for an incorrect reason.

129
Q

A swimmer went to a privately owned lake resort whose owner charged a fee for admission. The beach had a roped-in swimming area and large signs directing swimmers not to swim anywhere but within the ropes. The lifeguards regularly enforced this rule. The resort also rented canoes and rowboats to its patrons, who could take them anywhere on the lake. The swimmer and two of his friends had rented a canoe and started to paddle out toward the other side of the lake when the swimmer saw a volleyball game starting on the beach that he wanted to join. He left his friends in the canoe and started swimming to shore. He was only a few yards outside of the roped-in swimming area when he started, but he angled away from the swimming area toward the area of the beach where the volleyball net was set up. Although the lifeguard on duty saw him, she did not warn him to return to the swimming area. When the depth of the water was about four feet, he put his foot down and was severely cut by the jagged edge of a rusted metal stake protruding a few inches out of the bottom of the lake. The swimmer had not seen the stake even though the water was clear and it was visible if he had looked down.If the swimmer sues the resort for his injury in a jurisdiction that applies the traditional rules for landowners and possessors of land, is he likely to recover?

A No, because the stake could have been seen by the swimmer.
B No, because he was swimming outside of the roped-in area.
C Yes, because the lifeguard on duty saw him and did not warn him to return to the swimming area.
D Yes, because he is a public invitee of the resort.

A

B No, because he was swimming outside of the roped-in area.

The swimmer cannot recover from the resort because he did not have invitee status when he was injured. In jurisdictions following the traditional rules for landowners and possessors of land, the nature of a duty of an owner or occupier of land to those on the premises depends on the legal status of the plaintiff in regard to the property, i.e., whether the plaintiff is a trespasser, licensee, or invitee. An invitee is a person who enters onto the premises in response to an express or implied invitation of the landowner. Those who enter as members of the public for a purpose for which the land is held open to the public and those who enter for a purpose connected with the business or other interests of the landowner are considered invitees. However, a person will lose his status as an invitee if he exceeds the scope of the invitation—if he goes onto a portion of the property where his invitation cannot reasonably be said to extend. Here, the swimmer was an invitee of the resort in the areas to which it allowed its patrons to go. However, the resort clearly identified the boundaries of the area held open to swimmers, and the swimmer could not reasonably have believed that he was invited to swim in the area where he was injured. Because the swimmer was at most a licensee when he was injured, the resort did not owe him a duty to make reasonable inspections of that area to discover dangerous conditions and make them safe. At most, the resort had a duty only to warn the swimmer of known dangerous conditions that create an unreasonable risk of harm to him and that he is unlikely to discover, and nothing in the facts indicates that any employees of the resort knew of the stake under the water. The swimmer therefore cannot recover against the resort. (A) is not as good a choice as (B). While a landowner is not liable for a dangerous condition that is obvious to the entrant on the land, the fact that the stake was visible does not establish that it was obvious, given that the swimmer was looking forward rather than down. Whether a danger is obvious is determined by all of the surrounding circumstances, not just whether the danger is visible. The better reason why the swimmer cannot recover is because he was no longer an invitee. (C) is incorrect because the lifeguard’s failure to direct the swimmer to the swimming area would not constitute an invitation to swim in the restricted area; at most, it would establish only that the swimmer was a licensee rather than a trespasser when he swam in that area. A licensee is one who enters onto land with the possessor’s permission, express or implied, for his own purpose or business rather than for the possessor’s benefit. The lifeguard’s conduct may have constituted implied permission for the swimmer to exit the lake in a nonswimming area for his own benefit, but it does not establish that he reasonably believed that he was invited to swim in that area. (D) is incorrect because the swimmer lost his status as an invitee when he exceeded the scope of his invitation by swimming in an area where swimming was not permitted.

130
Q

A state statute required that any freight train operating within the city limits be able to stop within 200 yards of applying its brakes. No fixed speed limit was established or particular type of braking mechanism required, but through either lowered speed or braking power, the 200‑yard limit was required of all trains. Another statute prohibited vehicles from being within the railroad crossing when the lights on the warning signs are flashing or when the gates are lowered. One day, as a freight train was entering the city limits, the engineer saw a car stalled at a street crossing ahead. He immediately applied full braking power, but was unable to stop the train before it had hit and demolished the car. The driver of the car had gotten clear before the impact, but brought suit against the freight line for property damage to the $25,000 car. At trial, the parties stipulated that the car was stalled within the crossing while the warning lights were flashing. Evidence at trial established that the distance from the point at which the engineer applied the train’s brakes to the point of impact was 150 yards, and from the braking point to the point at which the train finally stopped was 225 yards. No other evidence of negligence was presented by the driver. At the end of the driver’s case, the freight line moved for a directed verdict.Should the court grant the motion?

A No, because the freight line was negligent per se.
B No, because the freight line was strictly liable for its violation of the braking statute.
C Yes, because the driver’s car was on the freight line’s tracks in violation of the crossing statute.
D Yes, because the freight line’s violation of the braking statute was not the cause in fact of the accident.

A

D Yes, because the freight line’s violation of the braking statute was not the cause in fact of the accident.

The court should grant the motion because the driver did not establish the cause‑in‑fact element of his prima facie case against the freight line. The primary test for cause in fact (actual cause) is the “but for” test: An act is the cause in fact of an injury when the injury would not have occurred but for the act. Even though the freight line had a duty created by the statute to be able to stop its train within 200 yards of first braking, and breached that duty (establishing the first two elements of the driver’s prima facie case), it must still be shown that the collision would not have occurred in the absence of the breach. Because the car was only 150 yards from the point of braking, even a train in compliance with the statute would have struck it. Since no other evidence of negligence has been presented, the motion should be granted. (A) is incorrect because establishing the freight line’s “negligence per se” through violation of the statute only establishes a conclusive presumption of duty and breach of duty; the plaintiff must still prove causation. (B) is incorrect because generally violation of a statute does not create strict liability; even if it did in this case, the plaintiff would still have to prove causation as part of the prima facie case for strict liability. (C) is not correct because the court will not reach the issue of the plaintiff’s contributory negligence in this case because the prima facie case for the defendant’s negligence has not been established. Furthermore, establishing the plaintiff’s contributory negligence by violation of a statute uses the same rules that govern whether a statute can establish the defendant’s negligence. Hence, the driver’s violation of the crossing statute may be excused if the trier of fact determines that compliance was beyond his control because his car stalled.

131
Q

A man was negligently driving down the road, not paying attention to where he was going. Because of this, he hit and seriously injured a pedestrian who was lawfully crossing the street. The accident was witnessed by a friend of the woman who was standing on the sidewalk. The friend suffered extreme emotional distress that physically affected her nervous system. The friend brings suit against the driver for negligent infliction of emotional distress in a jurisdiction that has adopted the majority approach in bystander cases.Will the friend prevail?

A Yes, because she witnessed the pedestrian being seriously injured by the driver.
B Yes, because severe shock to the nervous system constitutes a physical injury.
C No, because she was not crossing the street with the pedestrian.
D No, because she was not a close relative of the pedestrian.

A

D No, because she was not a close relative of the pedestrian.

The friend will lose because she was not a close relative of the pedestrian. If a bystander suffers distress from seeing injury to another, a majority of courts now allow recovery if (i) the plaintiff and the person injured by the defendant are closely related, (ii) the plaintiff was present at the scene of the injury, and (iii) the plaintiff personally observed or perceived the event. Here, the friend is not related to the pedestrian; hence, she cannot recover for her distress. (A) is incorrect because witnessing the injury to another is not sufficient. (B) is incorrect even though the friend did suffer physical symptoms from the distress. As discussed above, she has not met the requirements to recover. (C) is incorrect because the friend’s proximity to the pedestrian is only relevant in the minority of jurisdictions that have retained the “zone of danger” requirement for bystander recovery. Here, the jurisdiction has adopted the majority approach.

132
Q

A hiker in an isolated area encountered a cross-country skier who had broken her leg. The hiker created a makeshift sled and began pulling the skier to the nearest road. As the hiker was pulling her across the ice of a lake, the ice gave way and they went into the water. The hiker was unable to get out of the water and drowned. The skier was able to pull herself to shore and eventually was rescued. However, she suffered severe hypothermia and lost some of her toes to frostbite as a result of being in the water.Does the skier have a cause of action for damages against the hiker’s estate?

A No, because the hiker had no duty to come to the skier’s aid.
B No, because the hiker did not survive the accident.
C No, unless the hiker acted negligently in attempting to cross the ice.
D No, unless the hiker acted with gross negligence in his attempt to cross the ice.

A

C No, unless the hiker acted negligently in attempting to cross the ice.

The hiker’s estate may be liable to the skier if the hiker acted negligently when he was rescuing the skier. As a general rule, no legal duty is imposed upon any person to affirmatively act for the benefit of others. However, one who gratuitously acts for the benefit of another, although under no duty to do so in the first instance, is then under a duty to act like a reasonable person. Here, the hiker was under no duty to come to the skier’s assistance. Having done so, however, he was under a duty to use reasonable care in undertaking the rescue. If he acted negligently in doing so, he was in breach of his duty to the skier and the skier would have a cause of action against his estate. To prevail, the skier would also have to establish that her injuries would not have occurred but for the hiker’s negligent attempt to cross the ice, and that the skier herself was not at fault. In any case, the skier has a cause of action stemming from the hiker’s negligent conduct. Thus, (C) is correct and (A) is incorrect. (B) is incorrect. At common law, a tort action abated at the death of either the tortfeasor or the victim. However, most states have adopted survival statutes that change this result. Thus, the fact that the potential tortfeasor died would not preclude the skier from bringing an action against the tortfeasor’s estate. (D) is incorrect because the skier would have a cause of action even for the hiker’s ordinary negligence. Many states have “Good Samaritan” statutes that exempt those who gratuitously render emergency assistance from liability for other than gross negligence, but most of these statutes apply only to health care providers rendering emergency medical assistance. Therefore, the skier could recover even if the hiker’s negligence did not amount to gross negligence.

133
Q

In the course of repainting an apartment, the landlord of a small apartment building used a professional strength, stain-killing primer manufactured by a paint company for professional painters. The building’s common ventilation system was running as the landlord applied the primer, and some fumes from the primer went through the ventilation system into the apartment of the upstairs tenant, who suffered injuries to her eyes as a result. The warning label on the can, which the landlord read, stated: “Danger. This material is extremely hazardous and volatile. Do not use near open flame. Use only with adequate ventilation.” The product contained a chemical known to be harmful to people’s eyes, but in the 15 years that the product has been on the market, there were no reported cases of anyone suffering an eye injury from the product. However, professional painters routinely close off or shut down any common ventilation systems in buildings before using the product.If the tenant brings an action against the paint company on a theory of strict liability, will she recover?

A Yes, because the product was used as intended and she was injured thereby.
B Yes, because the label on the product did not warn of the risk of the fumes causing eye injury.
C No, because the fact that no one had previously been injured demonstrated that the warning label on the product was sufficient.
D No, because the landlord acted negligently by leaving the ventilation system on.

A

B Yes, because the label on the product did not warn of the risk of the fumes causing eye injury.

The tenant will likely prevail because the lack of a warning about eye injuries made the product unreasonably dangerous. A products liability action based on strict liability requires the following: (i) the defendant is a commercial supplier; (ii) the defendant produced or sold a defective product; (iii) the defective product was the actual and proximate cause of the plaintiff’s injury; and (iv) the plaintiff suffered damage to person or property. Here, the paint company is a commercial supplier of a “defective” product. Although the primer was not actually defective in that it apparently performed as it was meant to do, it is legally defective if it was unreasonably dangerous and could be made safer by adequate warnings. Here, the paint company knew of the danger and could easily have placed a specific warning on the label. Even though professional users may have known of the danger, it was not obvious, and it could have been avoided at minimal cost by including a specific warning. That would have alerted the landlord to the danger, making it more likely that he would take precautions that would have prevented the tenant from being injured. To prove actual cause where the plaintiff’s claim is that the product is defective because of lack of an adequate warning, the plaintiff is entitled to a presumption that an adequate warning would have been read and heeded. Thus, the tenant can likely establish liability on her cause of action. (A) is wrong because even in a strict liability action, liability will be found only if the product is defective, not just because someone was injured when it was used for its intended purpose. (C) is wrong because the manufacturer must warn of the danger, and its duty is not satisfied merely because there have been no injuries to date by following the instructions on the label. (D) is wrong because, given the inadequacy of the warnings, any negligence on the landlord’s part would be ordinary foreseeable negligence that would not cut off the paint company’s liability for its defective product.

134
Q

A homeowner purchased a riding lawn mower from a lawn mower dealer. During his first use of the mower, the homeowner noticed that the mower was vibrating when he turned, but he was able to finish mowing. A few days later, the homeowner lent the mower to his neighbor. The neighbor was driving the mower back to his yard when he made a turn and a wheel broke off, causing the neighbor to be thrown off the lawn mower and onto the sidewalk. The neighbor was injured.The neighbor brought a negligence action against the dealer for his injuries. At trial, the neighbor presented evidence that the wheel broke because of a manufacturing defect. The dealer presented evidence that the homeowner could have discovered the defect after the mower began vibrating when he used it for the first time.In this action, who is likely to prevail?

A The neighbor, because the lawn mower was sold by the dealer with an unreasonably dangerous defect.
B The neighbor, because the defect in the wheel would not likely have occurred in the absence of negligence.
C The dealer, because the homeowner should have discovered the defect when the mower first started vibrating.
D The dealer, because there is no evidence that the dealer had reason to know that the lawn mower was defective.

A

D The dealer, because there is no evidence that the dealer had reason to know that the lawn mower was defective.

The dealer will prevail because there is no evidence that it should have discovered the defect. To prove breach of duty in a products liability action based on negligence, the plaintiff must show (i) negligent conduct by the defendant leading to (ii) the supplying of a defective product by the defendant. However, a dealer who buys from a reputable manufacturer with no reason to anticipate that the product is dangerous need make only a cursory inspection of the goods to avoid liability for manufacturing defects. Here, there is no evidence that the dealer should have known that the wheel was defective; hence the dealer will likely prevail. (A) is incorrect because merely selling the lawn mower with an unreasonably dangerous defect, without knowing or being expected to know of the defect, will not subject the dealer to liability for negligence. The statement in (A) is more appropriate in an action based on strict liability. (B) is incorrect because the use of res ipsa loquitur suggested by that choice would be directed at the manufacturer rather than the dealer, and the dealer is not liable for the manufacturer’s negligence. (C) is incorrect because the negligent failure of an intermediary to discover a defect is not a superseding cause. If the dealer were otherwise liable, the negligent failure of the homeowner to discover the defect in the wheel would not cut off the dealer’s liability.

135
Q

A ballplayer became ill soon after consuming sunflower seeds marketed by a farm products company. The package of seeds was inspected and foreign matter was discovered on the seeds.If the ballplayer brings an action against the farm products company on the basis of strict tort liability, which of the following would be most helpful for the company to avoid liability?

A The foreign matter on the seeds was a rare mold that could not be detected by tests commonly used for establishing that sunflower seeds are safe for human consumption.
B The seeds were sold in their natural state, and had not been manufactured or processed by the farm products company in any way.
C In answer to an interrogatory, the ballplayer has acknowledged that he has no evidence that his illness was caused by the foreign matter on the seeds.
D Although marketed under the label of the farm products company, the seeds had been collected and packaged for distribution by another company and any foreign matter on the seeds was the other company’s fault.

A

C In answer to an interrogatory, the ballplayer has acknowledged that he has no evidence that his illness was caused by the foreign matter on the seeds.

The ballplayer’s lack of evidence of causation is most helpful to the farm products company. One of the elements of a prima facie case for products liability based on strict liability is causation of some harm to the plaintiff by a defective product. The ballplayer must show that the farm products company is strictly liable as a commercial supplier of the seeds, and that the farm products company marketed a defective product. In addition, the defect must have actually and proximately caused some harm to the plaintiff, and there must be damages. If, as (C) states, the ballplayer can produce no evidence that the illness he suffered was caused by the seeds’ foreign matter, then he cannot prove the element of causation. Absent causation, a cause of action for strict liability will not lie. (A) is not as helpful to the farm products company as (C) because it does not preclude the ballplayer from establishing a prima facie case for strict liability. The fact that the foreign matter in the seeds was a rare mold might allow the farm products company to claim that it was not feasible to supply the seeds in a safer condition than they were (i.e., a “state of the art” defense), but the success of this argument is much less certain than the argument of no causation raised by choice (C). (B) is incorrect because the farm products company is strictly liable as a commercial supplier to refrain from selling a defective product. There is no requirement that the defendant in a strict liability action have manufactured or processed the product, only that the defendant be a commercial supplier of the product. The farm products company is a commercial supplier of the seeds by marketing them in its packaging. Therefore, the farm products company can be strictly liable even if the seeds were sold in their natural state. (D) is incorrect because, even if the seeds were actually collected and packaged by another company, the farm products company also is strictly liable as the company that markets the seeds and thus is part of the distributive chain.

136
Q

A company that owned a tract of land believed to be rich in mineral deposits contracted with a licensed excavator for the removal of soil from the property and delivery of the soil to the company’s laboratories. While one of the excavator’s trucks was on the way to the laboratory, the rear gate broke loose, dumping three tons of soil onto the highway. A motorist who was driving a short but safe distance behind the truck was unable to stop in time and collided with the soil, causing her serious injury. The rear gate had been negligently secured by one of the excavator’s employees.If the motorist sues the company for his injuries and does not prevail, what is the most likely reason?

A The rear gate was secured by the excavator’s employee.
B The excavator had a license to transport soil on the highway.
C The company’s duty in respect to the movement of its soil on the highway was delegable.
D The transportation of soil on the highways was a common practice in the area where the accident occurred.

A

C The company’s duty in respect to the movement of its soil on the highway was delegable.

The strongest basis for the motorist not prevailing is the absence of a nondelegable duty. The general rule is that a principal will not be liable for tortious acts of its agent if the agent is an independent contractor. However, a broad exception will impose liability on the principal if the duty is nondelegable because of public policy considerations. As long as the company was not subject to a nondelegable duty, it would not be liable for the negligence of the excavator’s employee in the transportation of its soil. (A) is not as good an answer as (C) because the fact that the accident was caused by the negligence of the independent contractor’s employee does not necessarily excuse the company from liability. (C) supplies the additional factor that enables the company to avoid liability. (B) is incorrect because the possession of a license by the excavator would not excuse the company from liability. (D) is incorrect because the fact that the transportation of soil was common to the area is relevant only for a strict liability action for abnormally dangerous activities, and the transport of soil by truck is not such an activity.

137
Q

A construction company that was putting in a swimming pool for a homeowner left a couple of large pieces of equipment in the backyard overnight. The equipment was not owned by the construction company but was leased from an equipment company, which was responsible for its repair and maintenance. After the workers had left, a seven-year-old boy came onto the homeowner’s property to play. The homeowner was aware that the boy often came onto his property to play with his dog. The boy climbed up on one of the pieces of equipment and began pushing buttons and moving levers. The engine started and the equipment began to move because the equipment company had not replaced a defective safety locking device on the ignition. The boy became frightened and jumped off, falling into the hole that had been dug that day, and was injured.The boy’s parents brought suit against the homeowner and the construction company.If the construction company is held liable for the boy’s injuries, may it recover anything from other parties?

A It may obtain indemnity from the equipment company because the equipment was negligently maintained in an unsafe condition.
B It may obtain contribution from the equipment company because the equipment was negligently maintained in an unsafe condition.
C It may obtain indemnity from both the equipment company and the homeowner
D It may not recover any damages it paid from any other party.

A

B It may obtain contribution from the equipment company because the equipment was negligently maintained in an unsafe condition.

Because the equipment company negligently maintained the equipment, the construction company could obtain contribution from the equipment company. When two or more tortious acts combine to proximately cause an indivisible injury to a plaintiff, each tortious actor will be jointly and severally liable for that injury. Joint and several liability permits a plaintiff to recover the entire judgment amount from any defendant. Contribution allows a defendant required to pay more than his share of damages to recover from the other jointly liable parties for the excess. In other words, contribution apportions responsibility among those who are at fault. Here, if the construction company is held liable for the boy’s injuries, it will be because of its negligence in leaving unattended a piece of equipment without a working safety locking device. However, because the equipment company, which was responsible for repair and maintenance of the equipment, negligently performed such maintenance, resulting in the absence of a working safety locking device, then the equipment company’s negligence would have combined with that of the construction company to proximately cause the boy’s injuries. This would render the companies jointly and severally liable to the boy for the entire damage incurred. Thus, if the construction company is held liable for the injuries, it has a claim against the equipment company, as a jointly liable party, for the amount it pays in excess of its share of damages. (A) is incorrect because indemnity is not available here. Indemnity involves shifting the entire loss between or among tortfeasors, and is available where: (i) there is a contractual promise to indemnify; (ii) there is a special relationship between the defendants that would allow for vicarious liability; or (iii) the defendant is a supplier in a strict products liability case who is liable to an injured customer, thus giving the supplier a right of indemnification against previous suppliers in the distribution chain. In addition, some states allow a joint tortfeasor to recover indemnification from a co-joint tortfeasor where there is a considerable difference in degree of fault. Here, there is no evidence of a contractual right to indemnity between the construction company and the equipment company, there is no relationship between them that causes the construction company to be held vicariously liable for the equipment company’s negligence, and this is not a strict products liability case. Also, there is no indication of a considerable difference in degree of fault between the two companies. Therefore, none of the circumstances in which indemnity is available is present. (C) is incorrect because it would allow for indemnity in this situation and, as explained above, the circumstances allowing for indemnity are simply not present here. (D) is incorrect because, as explained above, the construction company can recover from the equipment company based on contribution rules.

138
Q

Jon Snow testifies in court about a transaction between he and Tormund Giantsbane. Tormund later brings a defamation claim against Jon. What is Jon’s best defense?

A

Statements made by judges, jurors, counsel, witnesses, and/or parties in judicial proceedings are absolutely privileged.

139
Q

Company loans Don Draper a compay car to take his family on a vacation. Don drives like an asshole, and injures Plaintiff. A “permissive use” statute allows Plaintiff to recover from Company for Don’s negligence up to $25,000. If the state follows traditional contribution rules, can the Company get contribution from Don?

A

The Company can get full indemnity for all $25,000 because Don was the one who was actually at fault. The Company did not do anything wrong. Contribution is appropriate if the damages should be apportioned, but here, Don is 100% at fault. The only reason the Company had to pay anything was because of the statute.

140
Q

May a landowner use a vicious dog to protect his property?

A

No. Landowner will be liable for any resulting harm because protecting your property is only ok if it is proportional, and death or serious injury simply for trespassing is not.

141
Q

In an action against a doctor for wrongful pregnancy, what type of damages is a mother least likely to be recovered?

(a) medical expenses and pain and suffering from labor.
(b) the future cost of raising a child.
(c) the additional medical expenses to treat a genetic disease, the prevention of which was what gave rise to the doctor treating the mother.
(d) lost future earnings because of a disability suffered during labor.

A

(b)

142
Q

What is interference with business relations?

A

Elements:

1) Valid contractual relationship between P and a third party or valid business expectancy of P;
2) D’s knowledge of the relationship or expectancy
3) Intentional interference by D that induces a breach or termination of the relationship or expectancy; and
4) Damage to P

P has a cause of action against Ds for interference with probable future business relationships, for which P has a reasonable expectation of a benefit.

D’s action could be privileged if he was attempting to obtain business for himself and the relationship between the third party and P was only prospective.

Whether D is an independent contractor does not necessarily matter if he used improper means.

143
Q

If an employer loses a judgment to P for the negligent conduct of an employee, can the employer go after the employee for indemnification?

A

Indemnity is available in vicarious liability situations where one party is held liable for damages caused by an employee simply because of his relationship as the employee’s employer. See question 6 of Torts Set 3.

144
Q

If a taxi cab driver gets in an accident because a mechanic at a national auto repair chain negligently repaired the breaks, is the owner of the cab company liable for a passenger’s injuries?

A

Yes because the duty to repair the cab is non-delegable for common carriers so the mechanic’s negligence is attributable to the cab company owner because he cannot now provide a safe rides.

145
Q

What is intentional misrepresentation?

A

(i) Misrep by D;
(ii) Scienter (intent);
(iii) intent to induce P’s reliance;
(iv) causation (actual reliance);
(v) reliance was justified; and
(vi) damages.

146
Q

A man goes to a haunted house that warns patrons about the “scares” within both with signs at the premises and on the ticket. The man gets so scared when an actor playing a monster pops out that he jumps through a plate glass window. Can the man recover in an action for his injuries?

A

No, he assumed the risk.

147
Q

A little girl hops a fence at a closed amusement park near a residential neighborhood. She read and understood the no trespassing sign, but trespassed anyway. While playing on a ride, she cut herself on an exposed piece of metal, suffering a severe injury. If the company moves for summary judgment based on these facts, will it be granted?

A

No because even though she understood the sign, there is no evidence that she could understand the danger, and a jury could find that the company should have foreseen that children would sneak onto the property.

148
Q

What duty of care is owed by a bailor/bailee?

A

Bailor

a) if bailment is for the sole benefit of the bailee, then bailor must warn the bailee of known dangerous defects.
b) if bailment is for the sole benefit of the bailor, then bailor must warn the bailee of know dangerous defects, or defects that could have been discovered through reasonable diligence.

Bailee

a) if bailment is for the sole benefit of the bailor, then bailee is only liable for gross negligence.
b) if bailment is for the sole benefit of the bailee, then bailee is liable for even slight negligence.

Mutual beneficial bailments require the ordinary standard of care.

149
Q

Do Simulated Exam Question 17

A

See Answer